Free . Regular . Quality€¦ · 1. Currency notes are legal tenders 2. Currency notes are...

65
Free . Regular . Quality PRELIMS TEST SERIES - 2020 Full Length Test -1 www.YouTube.com/SleepyClasses

Transcript of Free . Regular . Quality€¦ · 1. Currency notes are legal tenders 2. Currency notes are...

Page 1: Free . Regular . Quality€¦ · 1. Currency notes are legal tenders 2. Currency notes are unlimited legal tenders 3. Currency notes are guaranteed by the Central Government 4. Currency

Free . Regular . Quality

PRELIMS TEST

SERIES - 2020

Full Length Test -1

www.YouTube.com/SleepyClasses

Page 2: Free . Regular . Quality€¦ · 1. Currency notes are legal tenders 2. Currency notes are unlimited legal tenders 3. Currency notes are guaranteed by the Central Government 4. Currency
Page 3: Free . Regular . Quality€¦ · 1. Currency notes are legal tenders 2. Currency notes are unlimited legal tenders 3. Currency notes are guaranteed by the Central Government 4. Currency

Free . Regular . Quality

www.courses.sleepyclasses.com

Test No._______________ Topic: ________________________________________

Syllabus: ____________________________________________________________

____________________________________________________________________

Scan QR Codeto download the

Android App.

Page 4: Free . Regular . Quality€¦ · 1. Currency notes are legal tenders 2. Currency notes are unlimited legal tenders 3. Currency notes are guaranteed by the Central Government 4. Currency

www.YouTube.com/SleepyClasses

www.sleepyclasses.com/

Full Length Test I

1. Operation Clean Art has been conceived

to crackdown the smuggling of which of the following species?

A. Tiger

B. Pangolin

C. Mongoose

D. None of the above

2. Consider the following states

1. Madhya Pradesh

2. Maharashtra

3. Arunachal Pradesh

4. Mizoram

Arrange the above in the increasing order of forest cover in India

A. 4 – 2 – 1 – 3

B. 2 – 4 - 1 – 3

C. 2 – 4 - 3 – 1

D. 4 – 2 – 3 – 1

3. Dnyanganga Wildlife Sanctuary is located in which of the following states?

A. Uttar pradesh

B. Maharashtra

C. Madhya Pradesh

D. None of the above

4. Consider the following Tiger Reserves

1. Dampa

2. Namdapha

3. Nameri

4. Pakke

Which of the above is/are located in Arunachal Pradesh?

A. 1, 2 and 4 only

B. 2, 3 and 4 only

C. 2 and 4 only

D. 1 and 3 only

5. Consider the following statements regarding CMS COP 13

1. The theme of CMS COP-13 is

‘Migratory species connect the planet and we welcome them home’.

2. India hosted the CMS COP for the first time.

3. The CMS COP 13 took place in Delhi.

Which of the above statements is/are correct?

A. 1 and 2 only

B. 2 and 3 only

C. 1 and 3 only

D. All of the above

6. Consider the following statements regarding Great Horn-bill

1. It is the State bird of both Arunachal Pradesh and Kerala.

2. Its IUCN status is near threatened.

Which of the above is/are correct?

A. 1 only

B. 2 only

C. Both 1 and 2

D. Neither 1 nor 2

7. Consider the following statements Global Tiger Forum

Page 5: Free . Regular . Quality€¦ · 1. Currency notes are legal tenders 2. Currency notes are unlimited legal tenders 3. Currency notes are guaranteed by the Central Government 4. Currency

www.YouTube.com/SleepyClasses

www.sleepyclasses.com/

1. It is the only intergovernmental international body established with members from willing countries to embark on a global campaign to protect the Tiger.

2. It was formed in 1994 with its secretariat at New Delhi.

Which of the above statements is/are correct?

A. 1 only

B. 2 only

C. Both 1 and 2

D. Neither 1 nor 2

8. Which of the following correctly defines Methanotrophs?

A. They are the bacteria responsible for producing methane

B. They metabolise and convert methane into carbon-dioxide

C. They have a negative effect on the growth of the rice

D. None of the above

9. Consider the following animals

1. Sikkim – Red Panda

2. Clouded Leopard - Mizoram

3. Sangai - Manipur

4. Assam – One Horned Rhino

Which of the above state animals are correctly matched?

A. 1, 3 and 4 only

B. 1, 2 and 4 only

C. 2, 3 and 4 only

D. All of the above

10. Which of the following is/are initiatives launched under National Mission on Sustainable Habitat?

1. The Energy Conservation Building Code

2. Urban Waste Management

3. Shift to public transport

Choose the correct option from below

A. 1 and 2 only

B. 2 and 3 only

C. 1 and 3 only

D. All of the above

11. Which of the following statements about PAT is/are true?

1. The PAT (Perform, Achieve and Trade) programme was a part of India’s National Mission on Enhanced Energy Efficiency.

2. It put in place a market-based mechanism to enhance cost effectiveness of improvements in energy efficiency in energy-intensive large industries and facilities.

3. One PAT cycle lasts five years, during which every plant – including the world’s best – is required to improve its specific energy consumption (SEC)

Choose the correct option from below

A. 1 and 2 only

B. 2 and 3 only

C. 1 and 3 only

D. All of the above

12. Which of the following adds/add nitrogen to the soil?

1. Excretion of urea by animals

2. Burning of coal by man

3. Death of vegetation

Select the correct answer using the codes given below.

Page 6: Free . Regular . Quality€¦ · 1. Currency notes are legal tenders 2. Currency notes are unlimited legal tenders 3. Currency notes are guaranteed by the Central Government 4. Currency

www.YouTube.com/SleepyClasses

www.sleepyclasses.com/

A. 1 and 2 only

B. 1 and 3 only

C. 2 and 3 only

D. All of the above

13. Consider the following statements regarding food chain

1. Grazing food chain is major conduit of energy for aquatic ecosystem.

2. In terrestrial ecosystem, large fraction of energy flows through detritus food chain.

3. Primary consumers belong to first trophic level

4. Detritus food chain begins with dead organic matter

Which of the above statements is/are correct?

A. 1, 2 and 3 only

B. 2, 3 and 4 only

C. 1, 2 and 4 only

D. All of the above

14. According to ISFR 2019, which of the following forest type covers maximum geographical area in India?

A. Tropical moist evergreen forests

B. Tropical moist deciduous forests

C. Tropical dry deciduous forests

D. Tropical dry evergreen forests

15. Consider the following

1. National Parks

2. Wildlife Sanctuaries

3. Biosphere Reserves

4. Conservation Reserves

5. Community Reserves

6. Tiger Reserves

Which of the above are not related with the Wildlife Protection Act?

A. 4 and 5 only

B. 3 only

C. 4, 5 and 6 only

D. None of the above

16. Which of the following is/are true in context of World Wildlife Fund?

1. It was established by United Nations as an international fundraising organization to work in collaboration

with existing conservation groups.

2. CITES was drafted as a result of a resolution adopted in 1963 at a meeting of members of WWF.

Select the Correct Option

A. 1 only

B. 2 only

C. Both 1 and 2

D. Neither 1 nor 2

17. Which of the following is/are covered under the Earth Summit of 1992?

1. Rio Declaration on Environment and Development

2. Agenda 21

3. Cartagena Protocol on Biosafety

Choose the correct option

A. 1 and 2 only

B. 1 and 3 only

C. 2 and 3 only

D. All of the above

18. The visit of foreigners in India to see the various places/events in the

Page 7: Free . Regular . Quality€¦ · 1. Currency notes are legal tenders 2. Currency notes are unlimited legal tenders 3. Currency notes are guaranteed by the Central Government 4. Currency

www.YouTube.com/SleepyClasses

www.sleepyclasses.com/

country, amounts to which of the following in terms of economy:

A. Production

B. Consumption

C. Import

D. Export

19. Which of the following are counted in India’s GDP calculation?

1. Informal sector activity

2. Re-exports

Select the incorrect answer using the code given below:

A. 1 only

B. 2 only

C. Both 1 and 2

D. Neither 1 nor 2

20. Consider the statements regarding the various inflation indices published in the country:

1. Wholesale Price Index (WPI) does not represent the inflation in services

2. Consumer Price Index (CPI) represents the inflation in goods and services

3. CPI and WPI represent the inflation of imported goods also

4. GDP deflator captures the inflation of the goods and services produced domestically

Select the correct answer using the code given below:

A. 1 and 2 only

B. 1, 2 and 3 only

C. 2, 3 and 4 only

D. All of the above

21. The decrease in dependency ratio (ratio of dependent population to working-age population) of a country may lead to which of the following situation:

1. Increase in the savings rate

2. The decrease in the savings rate

3. Increase in Capital Formation

4. The decrease in Capital Formation

Select the incorrect answer using the code is given below:

A. 1 and 3 only

B. 2 and 4 only

C. 1 and 4 only

D. 2 and 3 only

22. According to the latest budget document, India is planning to become a $5 Trillion Economy by 2024-25. Consider the following statements.

1. It is in nominal terms

2. It is in PPP terms

3. It will require compounded annual real growth of around 8%, with 4% inflation

Select the correct answer using the code is given below:

A. 1 only

B. 2 only

C. 1 and 3 only

D. 2 and 3 only

23. Consider the following statements regarding Incremental Capital Output Ratio (ICOR):

1. It shows how efficiently capital is being used to produce output

2. It is the extra unit of capital required to produce one additional unit of output

Page 8: Free . Regular . Quality€¦ · 1. Currency notes are legal tenders 2. Currency notes are unlimited legal tenders 3. Currency notes are guaranteed by the Central Government 4. Currency

www.YouTube.com/SleepyClasses

www.sleepyclasses.com/

3. It is the extra unit of output produced from one additional unit of capital

4. It is the ratio of change in capital to change in output

Select the incorrect answer using the code given below:

A. 3 only

B. 1 and 2 only

C. 1, 2 and 4 only

D. 1, 3 and 4 only

24. A country is going through a phase of industrialization. Which of the following statements are correct?

A. Capital to labour ratio increases

B. The productivity of labour increases

C. Total factor productivity increases

D. All of the above

25. Consider the following statement with reference to ‘Income Elasticity of Demand’:

1. It measures the responsiveness of demand for a particular good to changes in consumer income.

2. Using this concept, it is possible to tell if a particular good represents a necessity or a luxury.

Which of the statements given above is/are correct?

A. 1 only

B. 2 only

C. Both 1 and 2

D. Neither 1 nor 2

26. Consider the following statements regarding the services trade of India:

1. The value of export of services is more than the value of export of merchandise exports.

2. India services trade is steadily in surplus in the last decade

Select the correct answer using the code given below:

A. 1 only

B. 2 only

C. Both 1 and 2

D. Neither (i) nor (ii)

27. If a country has ‘Pegged’ (fixed but

adjustable) exchange rate, then consider the following statements:

1. Inflation in the country may make its exports less competitive

2. If the country devalues its currency in proportion to the inflation then its exports may remain competitive

Select the incorrect answer using the code given below:

A. 1 only

B. 2 only

C. Both 1 and 2

D. Neither 1 nor 2

28. Which of the following investors/ agencies can purchase the government of India securities/bonds?

1. Reserve Bank of India

2. Portfolio Investors

3. Financial Institutions

Select the correct answer using the code is given below:

A. 1 only

B. 1 and 3 only

C. 3 only

Page 9: Free . Regular . Quality€¦ · 1. Currency notes are legal tenders 2. Currency notes are unlimited legal tenders 3. Currency notes are guaranteed by the Central Government 4. Currency

www.YouTube.com/SleepyClasses

www.sleepyclasses.com/

D. All of the above

29. Which of the following statements will be true if the inflation in the economy is increasing?

1. The bond price will decrease

2. Bondholders will loose

3. The yield on bonds will increase

Select the correct answer using the code given below:

A. 1 only

B. 1 and 2 only

C. 3 only

D. All of the above

30. Consider the following statements:

1. Currency notes are legal tenders

2. Currency notes are unlimited legal tenders

3. Currency notes are guaranteed by the Central Government

4. Currency notes are guaranteed by the RBI

Select the correct answer using the code given below:

A. 1 and 3 only

B. 1 and 4 only

C. 1, 2 and 3 only

D. 2 and 4 only

31. Consider the following statements regarding the insurance cover provided to depositors by Deposit Insurance and Credit Guarantee Corporation (DICGC)

1. All commercial banks and urban cooperative banks have to register with DICGC for providing insurance to depositors

2. RBI incurs the insurance premium burden

3. Government incurs the insurance premium burden

Select the incorrect answer using the code is given below:

A. 1 only

B. 1 and 2 only

C. 3 only

D. 2 and 3 only

32. Which of the following were the reasons for the recent NBFC crisis in the economy?

1. Relying on short term financing to fund long-term investments

2. Asset liability mismatch (ALM)

3. Rollover risk of commercial papers

Select the correct answer using the code given below:

A. 1 and 3 only

B. 1 and 2 only

C. 2 and 3 only

D. All of the above

33. There is a Human Development Report (HDR) released by UNDP in December 2019. It talks about various kinds of inequalities. Consider the following statements in this regard

1. Climate change and technological transformation have the potential to deepen existing social and economic fault lines.

2. More men are free from gender bias as compared to a woman.

Which of the following statements are NOT true?

A. 1 only

Page 10: Free . Regular . Quality€¦ · 1. Currency notes are legal tenders 2. Currency notes are unlimited legal tenders 3. Currency notes are guaranteed by the Central Government 4. Currency

www.YouTube.com/SleepyClasses

www.sleepyclasses.com/

B. 2 only

C. Both 1 and 2

D. None of the above

34. Consider the following statements regarding the National Pension System

1. NPS has fixed interest rates decided by the Ministry of Finance.

2. A unique Permanent Retirement Account Number (PRAN) is generated and maintained by the Central Recordkeeping Agency (CRA) for individual subscribers.

3. NPS also has the facility of Tier-II account which is a voluntary account, which offers liquidity of investments and withdrawals.

4. On exit/retirement/superannuation, a minimum of 85% of the corpus is mandatorily utilized to procure a pension for life by purchasing an annuity from a life insurance company.

Which of the following statements are true?

A. 1 and 2 only

B. 2 and 3 only

C. 3 and 4 only

D. 1, 3 and 4 only

35. With reference to ‘voting in India’ which of the following statements is/are correct:

1. In India, Voting can be done in three ways: in person, through posts, and through a proxy.

2. Voting through post and proxy is only allowed to NRIs.

3. Lok Sabha recently passed Representation of the People (Amendment) Bill, 2017 to provide for an online voting facility to NRIs

Select the correct answer using the codes given below:

A. 1 only

B. 1 and 2 only

C. 1 and 3 only

D. 1,2 and 3

36. Consider the following statements about ‘Election Commissioners of India’:

1. The appointment of the chief election commissioner and other election commissioners shall be made by the president.

2. The election commissioners have equal powers and receive equal salary, allowances, and other perquisites, which are similar to those of a judge of the Supreme Court.

Which of the statements given above is/are correct?

A. 1 only

B. 2 only

C. Both 1 and 2

D. Neither 1 nor 2

37. Consider the following statements:

1. The Legislative Council of a State in India can be larger in size than half of the Legislative Assembly of that particular State.

2. The Governor of a State nominates the Chairman of the Legislative Council of that particular State.

3. The State Assembly can abolish a legislative council or create it on its own.

Which of the statements given above is/are incorrect?

A. 1 only

Page 11: Free . Regular . Quality€¦ · 1. Currency notes are legal tenders 2. Currency notes are unlimited legal tenders 3. Currency notes are guaranteed by the Central Government 4. Currency

www.YouTube.com/SleepyClasses

www.sleepyclasses.com/

B. 1 and 3 only

C. 1 and 2 only

D. 1, 2 and 3

38. Which of the following Directive Principle was not added by the 42nd

Amendment Act of 1976?

A. To secure opportunities for the healthy development of children (Article 39).

B. To promote equal justice and to provide free legal aid to the poor (Article 39 A).

C. To take steps to secure the participation

of workers in the management of industries (Article 43 A).

D. State to minimize inequalities in income, status, facilities, and opportunities (Article 38)

39. Consider the following statements about ‘National Commission for Scheduled Castes’:

1. It is a constitutional body.

2. NCSC has the power to look into complaints and welfare measures with regard to Scheduled Castes, backward classes, and Anglo-Indians.

3. The commission presents an annual report to the Parliament.

Which of the statements given above is/are correct?

A. 1 only

B. 1 and 2 only

C. 1 and 3 only

D. 2 and 3 only

40. Which of the following is/are a constitutional body?

1. National Human Rights Commission.

2. Central Information Commission

3. Union Public Service Commission

4. Finance Commission

Choose using the correct code:

A. 1 and 3 only

B. 1, 2 and 3 only

C. 3 only

D. 3 and 4 only

41. Consider the following statements regarding the Parliamentary Standing Committee on Finance:

1. The Chairman of the committee is nominated by the chairman of Rajya Sabha.

2. It consists of 31 members, 21 from Lok Sabha and 10 from Rajya Sabha.

Which of the above statement is correct?

A. 1 only

B. 2 only

C. Both 1 and 2

D. Neither 1 nor 2

42. Which committee of parliament considers and advises on matters concerning the affairs of the house, which do not fall

within the jurisdiction of any other parliamentary committee.

A. General Purpose Committee

B. Rules Committee of Lok Sabha

C. Business Advisory Committee

D. Committee on Subordinate legislation

43. Consider the following statements:

1. Indian Constitution does not have any provision for categorization of any state as a Special Category Status State.

Page 12: Free . Regular . Quality€¦ · 1. Currency notes are legal tenders 2. Currency notes are unlimited legal tenders 3. Currency notes are guaranteed by the Central Government 4. Currency

www.YouTube.com/SleepyClasses

www.sleepyclasses.com/

2. For special category status category states, the Centre pays 90 percent of the funds required in a centrally-sponsored scheme as against 60 percent in case of normal category states.

3. Low population density or the presence of a sizeable tribal population is one of the features required to get Special Category Status.

Which of the statements given above is/are correct?

A. 1 only

B. 1 and 3 only

C. 2 only

D. 1, 2 and 3

44. With reference to the provisions of ‘no-confidence motion’ consider the following statements:

1. The motion needs the support of one-third members to be admitted to the lower house.

2. It need not state the reasons for its adoption in the Lok Sabha.

3. It can be moved against an individual minister or a group of ministers or the entire council of ministers.

Which of the statements given above is/are incorrect?

A. 1 only

B. 1 and 3 only

C. 2 only

D. 1, 2 and 3

45. ‘NOTA’ option is available to voters in which of the following elections:

1. Lok Sabha elections

2. Rajya Sabha elections

3. State Assembly elections

4. State Council elections

Select the correct answer using the codes given below:

A. 1 and 3 only

B. 2 and 4 only

C. 1 only

D. 1, 2, 3, and 4

46. With reference to the constitutional provisions for the minority in India consider the following statement:

1. The Constitution of India does not define the word minority.

2. Gujarat became the first state in India to grant minority status for Jews.

3. Muslims, Christians, Sikhs, Zoroastrians, Buddhists, and Jains are the only communities to which the Union Government has accorded the status of the minority.

Which of the statements given above is/are correct?

A. 1 only

B. 1 and 3 only

C. 3 only

D. 1, 2 and 3

47. With reference to ‘North Eastern Council (NEC)’, consider the following statements

1. It is a statutory body with Governors and Chief Ministers of all the eight North Eastern States as its Member.

2. Minister of DoNER (Development of North Eastern Region) is ex-officio Chairman of the North Eastern Council (NEC).

Page 13: Free . Regular . Quality€¦ · 1. Currency notes are legal tenders 2. Currency notes are unlimited legal tenders 3. Currency notes are guaranteed by the Central Government 4. Currency

www.YouTube.com/SleepyClasses

www.sleepyclasses.com/

Which of the statements given above is/are correct?

A. 1 only

B. 2 only

C. Both 1 and 2

D. Neither 1 nor 2

48. With reference to ‘The Law Commission of India’, consider the

following statements:

1. The Law Commission of India is a statutory body constituted through the act of Parliament.

2. It works towards the progressive development and codification of the laws of the country.

Which of the statements given above is/are correct?

A. 1 only

B. 2 only

C. Both 1 and 2

D. Neither 1 nor 2

49. Consider the following statements about Deputy Chairman of Rajya Sabha:

1. In the election of Deputy Chairman of Rajya Sabha, members of both Lok Sabha and Rajya Sabha participate.

2. Deputy Chairman of Rajya Sabha is subordinate to Chairman of Rajya Sabha and is not directly responsible to Rajya Sabha.

Which of the above statement is Correct?

A. 1 only

B. 2 only

C. Both 1 and 2

D. Neither 1 nor 2

50. Consider the following statements:

1. A Supreme Court judge holds the office till the age of 62.

2. SC judge can only be removed on the ground of ‘misbehaviour’

3. SC judge is appointed by the President on the sole advice from Chief Justice of India.

Which of the statements given above is/are correct?

A. 1 and 2 only

B. 2 and 3 only

C. 2 only

D. None

51. Consider the following statement about the seismic wave:

1. P waves create crests and troughs while travelling through the earth.

2. S waves create compressions and expansions while travelling through earth.

3. S waves reach to the surface earlier than P waves.

Choose the correct answer from the following code:

A. 1 and 2 Only

B. 2 and 3 Only

C. All of the above

D. None of the above

52. Out of the following which one is not an

Intrusive volcanic landform?

A. Lacoliths

B. Sills

C. Cinder cone

D. Dyke

Page 14: Free . Regular . Quality€¦ · 1. Currency notes are legal tenders 2. Currency notes are unlimited legal tenders 3. Currency notes are guaranteed by the Central Government 4. Currency

www.YouTube.com/SleepyClasses

www.sleepyclasses.com/

53. Out of the following, which are formed by erosion

1. River terrace

2. Incised Meander

3. Fans

4. Deltas

Choose the correct answer from the following code:

A. 1 and 2 Only

B. 1,2 and 4 Only

C. 2,3 and 4 Only

D. 2 and 3 Only

54. Which of the given statement/statements is/are correct?

1. Metamorphic rock is a result of a transformation of a pre-existing rock.

2. Uplift and erosion are the two forces behind the formation of metamorphic rocks.

Choose the correct answer from the following Code:

A. 2 Only

B. None of the above

C. Both 1 and 2

D. 1 only

55. The temperature of a place is not affected

by:

A. Air Mass

B. Ocean currents

C. Longitude

D. Altitude

56. Which of the following is correct about the Laterite soils?

1. These soils develop in the area of low temperature and low rainfall.

2. These soils are rich in humus content.

3. These soils are highly infertile and poorly cultivated.

Which of the statements given above is/are correct?

A. 3 only

B. 2 and 3 only

C. 1 and 3 only

D. All of the above

57. Which of the following is not correct

about the air mass?

1. The air with distinctive characteristics in terms of temperature and humidity is called an air mass.

2. Air mass has little vertical variation in temperature and moisture.

3. The interaction of cold and warm air mass leads to rain.

Choose from the following code:

A. 1 Only

B. 2 only

C. 2 and 3 Only

D. All the above

58. Consider the statements:

1. Continental shelves contain lesser fossil fuel as compare to continental shelves.

2. Canyons and trenches are observed in the continental shelves

Choose the incorrect answer from the following code:

A. 1 Only

B. Both 1 and 2

C. 2 only

Page 15: Free . Regular . Quality€¦ · 1. Currency notes are legal tenders 2. Currency notes are unlimited legal tenders 3. Currency notes are guaranteed by the Central Government 4. Currency

www.YouTube.com/SleepyClasses

www.sleepyclasses.com/

D. None of the above

59. The ocean water temperature is affected by

1. Distribution of land and water

2. Prevailing wind

3. Ocean current

Choose the answer from the following code:

A. 2 and 3 Only

B. 1 and 3 Only

C. 1 and 2 Only

D. All the above

60. Consider the following statements about soil erosion.

1. Gully erosion takes place on level lands after a heavy shower and the soil

removal is not easily noticeable.

2. Sheet erosion is common on steep slopes.

3. A region with a large number of ravines is called badland topography.

Which of the given statements is/are correct?

A. 2 Only

B. 1 and 2 only

C. 2 and 3 only

D. None of the above

61. The increasing order of salinity among

the following water bodies is

1. Dead sea

2. Red sea

3. Atlantic Ocean

4. Black Sea

Choose the correct answer from the following code:

A. 4<3<2<1

B. 3<4<1<2

C. 4<3<2<1

D. 3<2<4<1

62. Consider the following statement about sugarcane crop:

1. It requires a hot and humid climate.

2. Too heavy rainfall results in high sugar content.

3. Short cool dry winter season during ripening and harvesting is ideal.

4. South India offers favourable climatic conditions for the growth of sugarcane.

Choose the correct statements

A. 1, 2 and 4 only

B. 2, 3 and 4 only

C. 1, 3 and 4 only

D. All of the above

63. The Black soils lack in:

1. Phosphorous

2. Nitrogen

3. Lime

4. Organic matter

Select the correct answer from the code given below:

A. 1, 3 and 4 only

B. 1 and 4 only

C. 1, 2, 3 and 4 only

D. 1, 2, and 4 only

64. Indian states/UTs which are the part of Cauvery basin are-

1. Tamil Nadu

2. Kerala

Page 16: Free . Regular . Quality€¦ · 1. Currency notes are legal tenders 2. Currency notes are unlimited legal tenders 3. Currency notes are guaranteed by the Central Government 4. Currency

www.YouTube.com/SleepyClasses

www.sleepyclasses.com/

3. Karnataka

4. Goa

Select the correct answer from the code given below:

A. 1, 2 and 4 only

B. 1, 2 and 3 only

C. 2, 3 and 4 only

D. 1, 2, 3 and 4

65. Consider the following state Forestry in India.

1. Agro-forestry is the raising of trees and agriculture crops on the same land inclusive of the waste patches.

2. Farm forestry is a process under which farmers grow trees for commercial non-commercial purposes on lands.

3. Social forestry means afforestation of barren lands with the purpose of helping in the environmental, social and rural development

Which of the given statements is/correct?

A. 3 only

B. 1 and 2 only

C. 1, 2 and 3

D. None of the above

66. The break-in ongoing monsoon is mainly caused by-

1. In the northern region, a break occurs

due to the absence of rain-bearing storm in the monsoon trough.

2. When winds run parallel to the western coast

Choose the correct answer from the following code:

A. 1 Only

B. 2 Only

C. Both 1 and 2

D. None of the above

67. Consider the following statements with respect to RAIDER – X

1. It has been developed by DRDO.

2. It is an explosive bomb.

Which of the above is/are correct?

A. 1 only

B. 2 only

C. Both

D. None

68. Which of the following points with respect to Coal Bed Methane (CBM) are correct

1. It is a conventional form of natural gas found in coal deposits and coal seams.

2. CBM is formed during the process of coalification, the transformation of plant material into coal.

Choose the correct statement

A. 1 only

B. 2 only

C. Both

D. None

69. Which of the following statement(s) is/are correct with respect to ExoMArs

1. It is a joint endeavor of ESA and Russian space agency, Roscosmos.

2. Its goal is to address the question of whether life has ever existed on Mars.

Choose the correct statement

A. 1 only

B. 2 only

Page 17: Free . Regular . Quality€¦ · 1. Currency notes are legal tenders 2. Currency notes are unlimited legal tenders 3. Currency notes are guaranteed by the Central Government 4. Currency

www.YouTube.com/SleepyClasses

www.sleepyclasses.com/

C. Both

D. None

70. Currently, India’s National Supercomputing Mission was in news, consider the following statements regarding the same

1. It is being implemented by department of science and technology (DST) and department of electronics and information technology (Diety).

2. It has produced the first supercomputer assembled indigenously called Param Shivay.

Choose the correct statement

A. 1 only

B. 2 only

C. Both

D. None

71. Recently a bio fortified crop by the name of MACS 4028 was developed in India, which of the following corresponds to the

same

A. Wheat

B. Rice

C. Maize

D. Ragi

72. Consider the following details with respect to the scheme SATHI (Sophisticated Analytical and technical

help institute)

1. It has been launched by the Ministry of HRD.

2. It aims to address the need for building shared, professionally managed and strong S&T infrastructure in the country which is readily accessible to academia, start-ups, manufacturing, industry and R&D labs, etc.

Choose the correct answer

A. 1 only

B. 2 only

C. Both

D. None

73. Consider the statements with respect to the process of “Reverse Osmosis”

1. It involves ‘a solvent (such as water) naturally moving from an area of low solute concentration, through a membrane, to an area of high solute concentration.

2. The problem associated with it is the deposition of brine.

3. Water purifiers use the above process of reverse osmosis.

Choose the correct answer

A. 1 and 2

B. 2 and 3

C. 1 and 3

D. All of the above

74. Consider the following points with respect to NASA Voyager spacecraft

1. It is the only probe ever to study Neptune and Uranus during planetary flybys.

2. It is the second manmade object to leave the heliosphere.

3. It is the only spacecraft to have visited all the 4 gas giant planets.

Choose the correct answer

A. 1 and 2

B. 2 and 3

C. 1 and 3

D. All of the above

Page 18: Free . Regular . Quality€¦ · 1. Currency notes are legal tenders 2. Currency notes are unlimited legal tenders 3. Currency notes are guaranteed by the Central Government 4. Currency

www.YouTube.com/SleepyClasses

www.sleepyclasses.com/

75. Consider the following points with respect to Aditya’s L1 mission

1. It is India’s first solar mission.

2. It will be launched using the GSLV in XL configuration.

3. The space-based observatory will have seven payloads.

Choose the correct answer

A. 1 and 2

B. 2 and 3

C. 1 and 3

D. All of the above

76. Which one of the following became a part of China in 1997 following the principle of “one country, two systems”?

A. Tibet

B. Hong kong

C. Xinjiang

D. Taiwan

77. Consider the following statements about ITBP – Indo Tibetan Border Police ITBP was raised in 1962.

1. It was basically a mountain raised forces.

2. It replaced Assam Rifles in Sikkim and Arunachal Pradesh in 2004-2005 for border guarding duty

Choose the correct answer

A. 1 and 2

B. 2 and 3

C. 1 and 3

D. All of the above

78. India has signed a Comprehensive Economic Partnership Agreement (CEPA) with

1. USA

2. Singapore

3. Japan

Choose the correct answer

A. 1 and 2

B. 2 and 3

C. 1 and 3

D. All of the above

79. Consider the following statements with respect to Hansen’s disease

1. It is caused by bacteria Mycobacterium laprae and mycobacterium lepromatosis.

2. It is non curable in nature.

3. It is generally common in poor people and transmitted by respiratory droplets.

Choose the correct answer

A. 1 and 2

B. 2 and 3

C. 1 and 3

D. All of the above

80. Hodeidah, a city in new recently, is

located in which of the following countries?

A. Syria

B. Iraq

C. Iran

D. Yemen

81. Which of the following statement(s) is/are correct in the context of

Chandrayaan 2

1. The Pragyan rover was designed to communicate with the Vikram lander,

Page 19: Free . Regular . Quality€¦ · 1. Currency notes are legal tenders 2. Currency notes are unlimited legal tenders 3. Currency notes are guaranteed by the Central Government 4. Currency

www.YouTube.com/SleepyClasses

www.sleepyclasses.com/

the lander with the orbiter and the orbiter with the Earth-based station

2. The lander was designed to function for 1 lunar day

Choose the correct statement

A. 1 only

B. 2 only

C. Both 1 and 2

D. Neither 1 nor 2

82. Which of the following best describes the term ‘Paraquat’?

A. It is a tribe that can recall the name of

their ancestors from 20 generations

B. Name of an iceberg which recently broke off in Antarctica

C. A herbicide which doesn’t have an antidote

D. Missiles used to attack oil facilities in Saudi Arabia

83. Consider the following statement about

the Harappan Agriculture system

1. The Bull was known to the people of Harappan Civilization

2. Evidence of plough field has been found at Harappan sites

3. Harappan agriculture was dependent on monsoon and no irrigation facility was required

Choose the correct answer from the following code:

A. 1 and 2 Only

B. 2 and 3 Only

C. All of the above

D. 1 and 3 Only

84. Consider the following about Mauryan Empire:

1. There was a uniform administrative system throughout the length and breadth of the Mauryan Empire

2. Asoka used the message of dhamma as a political tool to keep the large Mauryan Empire together

Choose the correct answer from the following code:

A. 1 Only

B. Both 1 and 2

C. 2 Only

D. None of the above

85. Arrange the following in chronological order with respect to their date of arrival in India?

1. Al-Biruni

2. Francois Bernier

3. Ibn Battuta

4. Marco Polo

Choose the correct answer from the following code:

A. 1<2<3<4

B. 1<4<3<2

C. 3<4<2<1

D. 3<2<4<1

86. Consider the following statements

1. Muhammad bin Tughlaq introduced the system of dhag and hullia as a part of military reform.

2. Alauddin Khalji created Diwan-i-Khairat to take care of orphans and widows

Choose the correct answer from the following code:

Page 20: Free . Regular . Quality€¦ · 1. Currency notes are legal tenders 2. Currency notes are unlimited legal tenders 3. Currency notes are guaranteed by the Central Government 4. Currency

www.YouTube.com/SleepyClasses

www.sleepyclasses.com/

A. 1 Only

B. Both 1 and 2

C. 2 Only

D. None of the above

87. Which of the following is not correct

about dahsala system?

A. Revenue was fixed on the average yield of the land assessed on the basis of the past 10 years.

B. The payment of revenue was generally made in cash.

C. The land was also divided into four categories Polaj, Milkiyat, Chachar, and Banjar.

D. The system was introduced by Todar Mal

88. Consider the following statement about Asokan edict:

1. Asoka was the first ruler to speak directly to his people through the inscriptions

2. They are found only in India

3. The inscription was written in Brahmi script, Kharosthi script and Greek script

Choose the correct answer from the following code:

A. 1 and 2 Only

B. 2 and 3 Only

C. All 1, 2 and 3

D. 1 and 3 Only

89. The Fort William College at Calcutta was founded during the reign of which of the following?

A. Sir John Shore

B. Lord Wellesley

C. Sir George Barlow

D. Lord Hastings

90. Consider the following statements about the Policy of Ring Fence:

1. It was aimed at creating buffer zones to defend the Company's frontiers.

2. The policy of subsidiary alliance was an extension of the Policy ring fence

3. Protection from Maratha and Afghans were the major reason for the Policy of Ring Fence.

Choose the answer from the following code:

A. 1 Only

B. 2 and 3 Only

C. 1 and 2 Only

D. All the above

91. Arrange the following in chronological order:

1. Anglo-Nepal War

2. Fourth Mysore War

3. First Afghan War

4. Second Maratha War

Choose the answer from the following code:

A. 2<4<1<3

B. 4<2<3<1

C. 3<4<2<1

D. 1<2<3<4

92. Consider the following statement about Revolt of 1857

1. The Revolt was sparked off by the episode of the greased cartridges

2. All the sections of Indian society supported the Revolt of 1857

Page 21: Free . Regular . Quality€¦ · 1. Currency notes are legal tenders 2. Currency notes are unlimited legal tenders 3. Currency notes are guaranteed by the Central Government 4. Currency

www.YouTube.com/SleepyClasses

www.sleepyclasses.com/

3. Hindu-Muslim divide was the major cause of failure of the Revolt

Choose the correct answer from the following code:

A. 1 Only

B. 1 and 2 Only

C. All the above

D. 2 and 3 Only

93. Consider the following statements:

1. The book ‘Sarvajanik Satyadharma’ was written by Jyotiba Phule

2. Social Service League was founded by

Narayan Malhar Joshi

3. The slogan ‘Back to the Vedas’ was given by Swami Vivekananda

Which of the statements given above is/are correct?

A. 1 and 3 only

B. 2 only

C. 1 and 2 only

D. 1, 2 and 3

94. With reference to the Indian Councils Act of 1892, consider the following statements:

1. It introduced an element of election for the first time

2. The budget could be discussed and voted upon

3. It provided for the non-official majority in the Imperial Legislative Council

Which of the statements given above is/are correct?

A. 1 only

B. 1 and 2 only

C. 2 and 3 only

D. 1, 2 and 3

95. Consider the following statements:

1. The demand of a full-fledged mass struggle with the goal of attaining Swaraj was approved by the Congress in its 1905 session

2. Swadeshi movement witnessed widespread participation from upper and middle-class Muslims

3. Traditional festivals were used to spread the propaganda of the Swadeshi movement

Which of the statements given above is/are incorrect?

A. 1 and 3 only

B. 1 and 2 only

C. 2 and 3 only

D. 1, 2 and 3

96. With reference to the revolutionary terrorist programme, consider the following statements:

1. Revolutionary terrorists aimed to create a violent mass revolution throughout the country.

2. Barrah dacoity was organized by Aurobindo Ghosh

3. Mitra Mela, a secret society, was organized by V.D Savarkar

Which of the statements given above is/are correct?

A. 1 and 2 only

B. 3 only

C. 2 and 3 only

D. 1, 2 and 3

97. With reference to Morley Minto reforms, consider the following statements:

Page 22: Free . Regular . Quality€¦ · 1. Currency notes are legal tenders 2. Currency notes are unlimited legal tenders 3. Currency notes are guaranteed by the Central Government 4. Currency

www.YouTube.com/SleepyClasses

www.sleepyclasses.com/

1. It introduced non-official majority in the Imperial Legislative Council

2. It provided for direct elections to the provincial legislature

3. Separate electorates were provided for the Muslims

Which of the statements given above is/are incorrect?

A. 1 and 3 only

B. 2 only

C. 1 and 2 only

D. 1, 2 and 3

98. With reference to the Ghadr programme, consider the following statements:

1. Their aim was to bring simultaneous revolt in all British colonies

2. It lacked an organized and sustained leadership

3. It preached militant nationalism with a completely secular approach

Which of the statements given above is/are correct?

A. 1 and 2 only

B. 1 and 3 only

C. 2 and 3 only

D. 1, 2 and 3

99. With reference to Home Rule League movement, consider the following statements:

1. Annie Besant set up her league before Tilak

2. Formation of linguistic state was one of the demands of the league set-up by Tila

3. Anglo-Indians and non-brahmins from the south did not join the Home Rule League

Which of the statements given above is/are correct?

A. 1 and 3 only

B. 2 and 3 only

C. 1 and 2 only

D. 1, 2 and 3

100. Which of the following demands were put forward by the Lucknow Pact?

A. Complete independence from the

British rule

B. All the members of the viceroy’s executive council should be Indians

C. Elected majority in the legislative councils

D. None of the above

Page 23: Free . Regular . Quality€¦ · 1. Currency notes are legal tenders 2. Currency notes are unlimited legal tenders 3. Currency notes are guaranteed by the Central Government 4. Currency

www.YouTube.com/SleepyClasses

www.sleepyclasses.com/

Full Length Test I

1. Operation Clean Art has been conceived to crackdown the smuggling of which of the following species?

A. Tiger

B. Pangolin

C. Mongoose

D. None of the above

Answer: C

Explanation:

Operation Clean Art - It was the first

pan India operation to crackdown on the smuggling of mongoose hair in the country. It was conceived by Wildlife Crime Control Bureau (WCCB) with the singular aim of ensuring that the mongoose hair brush trade should be closed down across the country.

The mongoose is listed in Schedule II Part 2 of the Wildlife Protection Act and any smuggling or possession of its body part is a non-bailable offence. Persons using brushes made of mongoose hair should be aware of it.

The reason why painters prefer brushes made of mongoose hair is because they are superior and hold colour better.

2. Consider the following states

1. Madhya Pradesh

2. Maharashtra

3. Arunachal Pradesh

4. Mizoram

Arrange the above in the increasing order of forest cover in India

A. 4 – 2 – 1 – 3

B. 2 – 4 - 1 – 3

C. 2 – 4 - 3 – 1

D. 4 – 2 – 3 – 1

Answer: D

Explanation: As per the ISFR 2019 report, largest forest covers in India:

Madhya Pradesh > Arunachal Pradesh > Chhattisgarh > Odisha > Maharashtra

Forest cover as percentage of total geographical area: Mizoram (85.41%) > Arunachal Pradesh (79.63%) > Meghalaya (76.33%) > Manipur (75.46%) > Nagaland (75.31%).

3. Dnyanganga Wildlife Sanctuary is located in which of the following states?

A. Uttar pradesh

B. Maharashtra

C. Madhya Pradesh

D. None of the above

Answer: B

Explanation:

The Dnyanganga Sanctuary is situated in the Buldhana district of Maharashtra. It is a part of the Melghat

Page 24: Free . Regular . Quality€¦ · 1. Currency notes are legal tenders 2. Currency notes are unlimited legal tenders 3. Currency notes are guaranteed by the Central Government 4. Currency

www.YouTube.com/SleepyClasses

www.sleepyclasses.com/

Tiger Reserve in Maharashtra. It is located near the Dnyanganga River which is a tributary of the Tapti River.

It was in news as recently a tiger travelled 1300 km distance between Maharashtra and Telangana passing through Dnyanganga Wildlife Sanctuary and Tipeshwar Tiger Reserve in Maharashtra.

4. Consider the following Tiger Reserves

1. Dampa

2. Namdapha

3. Nameri

4. Pakke

Which of the above is/are located in Arunachal pradesh?

A. 1, 2 and 4 only

B. 2, 3 and 4 only

C. 2 and 4 only

D. 1 and 3 only

Answer: C

Explanation: Dampa TR is in Mizoram. Nameri TR is in Assam. Rest two are in Arunachal pradesh.

Page 25: Free . Regular . Quality€¦ · 1. Currency notes are legal tenders 2. Currency notes are unlimited legal tenders 3. Currency notes are guaranteed by the Central Government 4. Currency

www.YouTube.com/SleepyClasses

www.sleepyclasses.com/

5. Consider the following statements regarding CMS COP 13

1. The theme of CMS COP-13 is ‘Migratory species connect the planet and we welcome them home’.

2. India hosted the CMS COP for the first time.

3. The CMS COP 13 took place in Delhi.

Which of the above statements is/are correct?

A. 1 and 2 only

B. 2 and 3 only

C. 1 and 3 only

D. All of the above

Answer: A

Explanation:

CMS COP-13 - India marks the beginning of super year of Biodiversity with the hosting of the 13th Conference of Parties (COP) of the Convention on the Conservation of Migratory Species of Wild Animals (CMS), an environmental treaty under the aegis of United Nations Environment Programme, from 17th to 22nd February 2020 at Gandhinagar in Gujarat. It is for the first time that India is hosting CMS COP.

The theme of CMS COP-13 is ‘Migratory species connect the planet and we welcome them home’. The CMS COP 13 logo is inspired by ‘Kolam’, a traditional artform from southern India. In the logo of CMS COP-13, Kolam art form is used to depict key migratory species in India like Amur falcon, humpback whale and marine turtles.

The mascot for CMS COP-13 is ‘Gibi – The Great Indian Bustard’. It is a critically endangered species and is listed in Schedule I under the Wildlife Protection Act, 1972.

6. Consider the following statements regarding Great Horn-bill

1. It is the State bird of both Arunachal Pradesh and Kerala.

2. Its IUCN status is near threatened.

Which of the above is/are correct?

A. 1 only

B. 2 only

C. Both 1 and 2

D. Neither 1 nor 2

Answer: A

Explanation:

It’s the State bird of both Arunachal Pradesh and Kerala. Sound of their take off is akin to the start of the steam locomotive of the Nilgiri Mountain Railway

Great Hornbill, commonly found in many parts of the Blue Mountains flying from one tree to another, may in a matter of time become extremely rare if habitat loss continues at the rate at which it has been happening for some time now.

The magnificent birds which were also a common sight in the evergreen rain forests of the Western Ghats are now forced, due to deforestation, to adapt themselves to hollows in silver oak trees which form part of thick coffee plantations in Singara and Moyar and some tea plantations in the lower part of the hills.

Pakke TR is known for its amazing sightings of four resident hornbill species.

Page 26: Free . Regular . Quality€¦ · 1. Currency notes are legal tenders 2. Currency notes are unlimited legal tenders 3. Currency notes are guaranteed by the Central Government 4. Currency

www.YouTube.com/SleepyClasses

www.sleepyclasses.com/

7. Consider the following statements Global Tiger Forum

1. It is the only intergovernmental international body established with members from willing countries to embark on a global campaign to protect the Tiger.

2. It was formed in 1994 with its secretariat at New Delhi.

Which of the above statements is/are correct?

A. 1 only

B. 2 only

C. Both 1 and 2

D. Neither 1 nor 2

Answer: C

Explanation:

The Global Tiger Forum (GTF) is an intergovernmental and international body established with members from willing countries to embark on a worldwide campaign, common approach, promotion of appropriate programmes and controls to save the remaining five sub-species of tigers in the wild distributed over 14 tiger range countries of the world.

Formed in 1994 with its secretariat at New Delhi, GTF is the only inter-

governmental & international body campaigning to save the TIGER worldwide. The General Assembly of GTF shall meet once in three years.

8. Which of the following correctly defines Methanotrophs?

A. They are the bacteria responsible for producing methane

B. They metabolise and convert methane into carbon-dioxide

C. They have a negative effect on the growth of the rice

D. None of the above

Answer: B

Explanation:

Scientists at Agharkar Research Institute (ARI), Pune, an autonomous

institute under the Department of Science & Technology, have isolated 45 different strains of methanotrophic bacteria which have been found to be capable of reducing methane emissions from rice plants.

Methanotrophs metabolise and convert methane into carbon-di-oxide. They can effectively reduce the emission of methane, which is the second most important greenhouse gas (GHG) and 26 times more potent as compared to carbon-di-oxide. In rice fields, methanotrophs are active near the roots or soil-water interfaces.

The team found that there was a decrease in methane emissions in inoculated plants with a positive or neutral effect on the growth of the rice. This could lead to the development of microbial inoculants for methane mitigation in rice.

Rice fields are human-made wetlands and are waterlogged for a considerable

Page 27: Free . Regular . Quality€¦ · 1. Currency notes are legal tenders 2. Currency notes are unlimited legal tenders 3. Currency notes are guaranteed by the Central Government 4. Currency

www.YouTube.com/SleepyClasses

www.sleepyclasses.com/

period. Anaerobic degradation of organic matter results in the generation of methane. Rice fields contribute to nearly 10% of global methane emissions. Very few studies in the world have focused on methanotrophs from tropical wetlands or tropical rice fields.

Besides methane mitigation studies, methanotrophs can also be used in methane value addition (valorization) studies. Bio-methane generated from waste can be used by the methanotrophs and can be converted to value-added products such as single-cell proteins, carotenoids, biodiesel, and so on.

The team is further working on methane valorization studies from the isolated methanotrophs. Such studies help reduce GHG emissions, especially anthropogenic or man-made emissions, which a pressing need in the age of global warming.

9. Consider the following animals

1. Sikkim – Red Panda

2. Clouded Leopard - Mizoram

3. Sangai - Manipur

4. Assam – One Horned Rhino

Which of the above state animals are correctly matched?

A. 1, 3 and 4 only

B. 1, 2 and 4 only

C. 2, 3 and 4 only

D. All of the above

Answer: A

Explanation:

Sikkim – Red Panda

Clouded Leopard – Meghalaya (Hence no. 2 is incorrect)

Sangai - Manipur

Assam – One Horned Rhino

10. Which of the following is/are initiatives launched under National Mission on Sustainable Habitat?

1. The Energy Conservation Building Code

2. Urban Waste Management

3. Shift to public transport

Choose the correct option from below

A. 1 and 2 only

B. 2 and 3 only

C. 1 and 3 only

D. All of the above

Answer: D

Explanation: The National Mission for Sustainable Habitat which is a component of the National Action Plan for Climate Change will broadly cover the following aspects –

Extension of the energy conservation building code - which addresses the design of new and large commercial buildings to optimize their energy demand;

Better urban planning and modal shift to public transport - make long term transport plans to facilitate the growth of medium and small cities in such a way that ensures efficient and convenient public transport;

Recycling of material and urban waste management - a special area of focus will be development of technology for producing power form waste.

Page 28: Free . Regular . Quality€¦ · 1. Currency notes are legal tenders 2. Currency notes are unlimited legal tenders 3. Currency notes are guaranteed by the Central Government 4. Currency

www.YouTube.com/SleepyClasses

www.sleepyclasses.com/

11. Which of the following statements about PAT is/are true?

1. The PAT (Perform, Achieve and Trade) programme was a part of India’s National Mission on Enhanced Energy Efficiency.

2. It put in place a market-based mechanism to enhance cost effectiveness of improvements in energy efficiency in energy-intensive large industries and facilities.

3. One PAT cycle lasts five years, during which every plant – including the world’s best – is required to improve its specific energy consumption (SEC)

Choose the correct option from below

A. 1 and 2 only

B. 2 and 3 only

C. 1 and 3 only

D. All of the above

Answer: A

Explanation:

The PAT (Perform, Achieve and Trade) programme was designed in response to the Indian government’s commitment in the National Action Plan on Climate Change (NAPCC) 2008.

As part of its National Mission on Enhanced Energy Efficiency, it was required to put into place a "market based mechanism to enhance cost effectiveness of improvements in energy efficiency in energy-intensive large industries and facilities, through certification of energy savings that could be traded."

The first PAT cycle was started in 31st March, 2012. Its rules and targets were notified the same month and amended in 2016.

One cycle lasts three years, during which every plant – including the world’s best – is required to improve its specific energy consumption (SEC), which is defined as ‘energy entering the factory gate over a year/material leaving the factory gate over the year.’

Plants with lower SECs were given lower targets and vice versa. However, the idea is to encourage industries to surpass their targets. Those exceeding their targets, thus, received Energy Savings Certificates (ESCerts) for excess savings that can be traded for compliance with plants that didn’t meet their targets, or banked for the next cycle.

12. Which of the following adds/add nitrogen to the soil?

1. Excretion of urea by animals

2. Burning of coal by man

3. Death of vegetation

Select the correct answer using the codes given below.

A. 1 and 2 only

B. 1 and 3 only

C. 2 and 3 only

D. All of the above

Answer: B

Explanation: Burning of coal adds sulphur oxide to the atmosphere.

13. Consider the following statements regarding food chain

1. Grazing food chain is major conduit of energy for aquatic ecosystem.

2. In terrestrial ecosystem, large fraction of energy flows through detritus food chain.

Page 29: Free . Regular . Quality€¦ · 1. Currency notes are legal tenders 2. Currency notes are unlimited legal tenders 3. Currency notes are guaranteed by the Central Government 4. Currency

www.YouTube.com/SleepyClasses

www.sleepyclasses.com/

3. Primary consumers belong to first trophic level

4. Detritus food chain begins with dead organic matter

Which of the above statements is/are correct?

A. 1, 2 and 3 only

B. 2, 3 and 4 only

C. 1, 2 and 4 only

D. All of the above

Answer: C

Explanation:

There are two types of food chains, namely grazing food chain and detritus food chain. Grazing food chain starts with producer followed by herbivores followed by primary carnivore and secondary carnivore. It ends with decomposers. It is the major conduit of

energy for aquatic ecosystem. Producers occupy the first trophic level.

The primary consumers occupy the second trophic level.

The secondary consumers occupy the third trophic level. Detritus food chain starts with dead organic matter and goes to detrivores. In terrestrial ecosystem, large fraction of energy

flows through detritus food chain.

14. According to ISFR 2019, which of the following forest type covers maximum geographical area in India?

A. Tropical moist evergreen forests

B. Tropical moist deciduous forests

C. Tropical dry deciduous forests

D. Tropical dry evergreen forests

Answer: C

Explanation: Tropical dry deciduous forests run from the Himalayan foot hills to Kanyakumari and comprise important trees like bijasal, teak, tendu, amaltas, khair, palas, rosewood and axlewood.

15. Consider the following

1. National Parks

2. Wildlife Sanctuaries

3. Biosphere Reserves

4. Conservation Reserves

5. Community Reserves

6. Tiger Reserves

Which of the above are not related with the Wildlife Protection Act?

A. 4 and 5 only

B. 3 only

C. 4, 5 and 6 only

D. None of the above

Answer: B

Explanation:

National Parks, Wildlife Sanctuaries, Conservation Reserves, Community Reserves and Tiger Reserves are established as per provisions of Wildlife Protection Act, there is no law as such under which Biosphere Reserves are established.

16. Which of the following is/are true in

context of World Wildlife Fund?

1. It was established by United Nations as an international fundraising organization to work in collaboration with existing conservation groups.

2. CITES was drafted as a result of a resolution adopted in 1963 at a meeting of members of WWF.

Page 30: Free . Regular . Quality€¦ · 1. Currency notes are legal tenders 2. Currency notes are unlimited legal tenders 3. Currency notes are guaranteed by the Central Government 4. Currency

www.YouTube.com/SleepyClasses

www.sleepyclasses.com/

Select the Correct Option

A. 1 only

B. 2 only

C. Both 1 and 2

D. Neither 1 nor 2

Answer: D

Explanation:

In 1961, a limited number of

organizations around the world—such as the International Union for the Conservation of Nature and Natural Resources (IUCN) and The Conservation Foundation—were trying to meet conservation needs, but were desperately short of funds.

The first call for broad support was the Morges Manifesto, signed in 1961 by 16 of the world’s leading conservationists. The Morges Manifesto stated that while the expertise to protect the world environment existed, the financial support to achieve this protection did not.

The decision was made to establish World Wildlife Fund as an international fundraising organization to work in collaboration with existing conservation groups and bring substantial financial support to the conservation movement on a worldwide scale.

CITES was drafted as a result of a resolution adopted in 1963 at a meeting of members of IUCN (The World Conservation Union).

17. Which of the following is/are covered under the Earth Summit of 1992?

1. Rio Declaration on Environment and Development

2. Agenda 21

3. Cartagena Protocol on Biosafety

Choose the correct option

A. 1 and 2 only

B. 1 and 3 only

C. 2 and 3 only

D. All of the above

Answer: A

Explanation:

The United Nations Conference on Environment and Development (UNCED), also known as the Rio de Janeiro Earth Summit, the Rio Summit, the Rio Conference, and the Earth Summit was a major United Nations conference held in Rio de Janeiro from 3 to 14 June 1992. The Earth Summit resulted in the following documents:

Rio Declaration on Environment and Development

Agenda 21

Forest Principles

18. The visit of foreigners in India to see the various places/events in the country, amounts to which of the following in terms of economy:

A. Production

B. Consumption

C. Import

D. Export

Answer: D

Explanation:

Exports mean produced within the country and sold to foreigners (or non-residents). If a foreigner is coming to India and then purchasing goods and services, then it is a case of exports.

Page 31: Free . Regular . Quality€¦ · 1. Currency notes are legal tenders 2. Currency notes are unlimited legal tenders 3. Currency notes are guaranteed by the Central Government 4. Currency

www.YouTube.com/SleepyClasses

www.sleepyclasses.com/

In the same way, if a foreigner is coming to India for medical treatment or tourism thenthe foreigner is basically purchasing medical and tourism services produced in our country

19. Which of the following are counted in India’s GDP calculation?

1. Informal sector activity

2. Re-exports

Select the incorrect answer using the code given below:

A. 1 only

B. 2 only

C. Both 1 and 2

D. Neither 1 nor 2

Answer: C

Explanation:

Informal economic activity constitutes around 30% of the GDP. We may not be able to measure it accurately but while calculating the GDP figures informal activity is extrapolated based on formal activities and is included in GDP.

Re-exports means, something imported and then processed and then again exported. So, whatever goes in processing will be part of our GDP.

20. Consider the statements regarding the various inflation indices published in the country:

1. Wholesale Price Index (WPI) does not represent the inflation in services

2. Consumer Price Index (CPI) represents the inflation in goods and services

3. CPI and WPI represent the inflation of imported goods also

4. GDP deflator captures the inflation of the goods and services produced domestically

Select the correct answer using the code given below:

A. 1 and 2 only

B. 1, 2 and 3 only

C. 2, 3 and 4 only

D. All of the above

Answer: D

Explanation:

Services are not traded/transacted in the wholesale markets. So, WPI data does not include inflation due to services.When goods are imported in India, first they move to the wholesale mandis and then they come in the retail markets.

So, wholesale prices and retail prices both get impacted because of imported goods.GDP Deflator (is) = Nominal GDP/Real GDP Since GDP includes only domestic goods and services, hence, GDP Deflator does not include inflation due to imported goods and services.

21. The decrease in dependency ratio (ratio of dependent population to working-age population) of a country may lead to which of the following situation:

1. Increase in the savings rate

2. The decrease in the savings rate

3. Increase in Capital Formation

4. The decrease in Capital Formation

Select the incorrect answer using the code is given below:

Page 32: Free . Regular . Quality€¦ · 1. Currency notes are legal tenders 2. Currency notes are unlimited legal tenders 3. Currency notes are guaranteed by the Central Government 4. Currency

www.YouTube.com/SleepyClasses

www.sleepyclasses.com/

A. 1 and 3 only

B. 2 and 4 only

C. 1 and 4 only

D. 2 and 3 only

Answer: B

Explanation:

Whenever in any country, the working population increases and the dependent population decreases, the savings in the economy increases.

(This also happens at the family level. If a family has more working members and fewer dependents then savings of

the family increases). The increased savings lead to an increase in investments.

22. According to the latest budget

document, India is planning to become a $5 Trillion Economy by 2024-25. Consider the following statements.

1. It is in nominal terms

2. It is in PPP terms

3. It will require compounded annual real growth of around 8%, with 4% inflation

Select the correct answer using the code is given below:

A. 1 only

B. 2 only

C. 1 and 3 only

D. 2 and 3 only

Answer: C

Explanation:

2018-19 2024-25

$2.7 Trillion $ 5 Trillion

(nominal GDP) (Nominal GDP) So, it requires 85% growth in six years, which comes down to around 12% compounded annual growth. This 12% is nominal growth which can be achieved with real growth of around 8% and inflation of around 4%.

23. Consider the following statements regarding Incremental Capital

Output Ratio (ICOR):

1. It shows how efficiently capital is being used to produce output

2. It is the extra unit of capital required to produce one additional unit of output

3. It is the extra unit of output produced from one additional unit of capital

4. It is the ratio of change in capital to change in output

Select the incorrect answer using the code given below:

A. 3 only

B. 1 and 2 only

C. 1, 2 and 4 only

D. 1, 3 and 4 only

Answer: A

Explanation:

ICOR represents how much extra unit of capital is required to produce one additional unit of output.

It basically represents the (inverse of)

efficiency of the new capital. If our ICOR is 5 and we want a growth of 8% in GDP then we will have to do 40% investment.

24. A country is going through a phase of industrialization. Which of the following statements are correct?

Page 33: Free . Regular . Quality€¦ · 1. Currency notes are legal tenders 2. Currency notes are unlimited legal tenders 3. Currency notes are guaranteed by the Central Government 4. Currency

www.YouTube.com/SleepyClasses

www.sleepyclasses.com/

A. Capital to labour ratio increases

B. The productivity of labour increases

C. Total factor productivity increases

D. All of the above

Answer: D

Explanation:

When a country goes through industrialization, it uses more capital and less labour comparatively or we can say labours are replaced by capital (machinery). That means the ratio of capital to labour increases sharply.

Industrialization also leads to an increase in the production of goods and services (with the same amount of labour or maybe less labour). So, production per labour also increases which means an increase in labour productivity.

25. Consider the following statement with reference to ‘Income Elasticity of Demand’:

1. It measures the responsiveness of demand for a particular good to changes in consumer income.

2. Using this concept, it is possible to tell if a particular good represents a necessity or a luxury.

Which of the statements given above is/are correct?

A. 1 only

B. 2 only

C. Both 1 and 2

D. Neither 1 nor 2

Answer: C

Explanation:

Income elasticity of demand is calculated as the ratio of the percentage

change in quantity demanded to the percentage change in income. It measures the responsiveness of the quantity demanded a good or service to a change in income.

If income elasticity of demand for a commodity is less than 1 that means that with a change in income, demand is not changing much, which means, it is a necessity good. If the elasticity of demand is greater than 1, it is a luxury good or a superior good.

26. Consider the following statements regarding the services trade of

India:

1. The value of export of services is more than the value of export of merchandise exports.

2. India services trade is steadily in surplus in the last decade

Select the correct answer using the code given below:

A. 1 only

B. 2 only

C. Both 1 and 2

D. Neither (i) nor (ii)

Answer: B

Explanation: India’s services exports are around 7.7% of GDP in 2018-19, while merchandise exports are around 12.1% of GDP. India’s services imports are around 4.6% of GDP in 2018-19. India’s services trade has been consistently in surplus for the last decade.

27. If a country has ‘Pegged’ (fixed but adjustable) exchange rate, then consider the following statements:

1. Inflation in the country may make its exports less competitive

Page 34: Free . Regular . Quality€¦ · 1. Currency notes are legal tenders 2. Currency notes are unlimited legal tenders 3. Currency notes are guaranteed by the Central Government 4. Currency

www.YouTube.com/SleepyClasses

www.sleepyclasses.com/

2. If the country devalues its currency in proportion to the inflation then its exports may remain competitive

Select the incorrect answer using the code given below:

A. 1 only

B. 2 only

C. Both 1 and 2

D. Neither 1 nor 2

Answer: D

Explanation:

The pegged exchange rate means a country fixes its exchange rate with another country currency or a basket of currencies and when required changes it accordingly.

In the case of inflation in the domestic market the price of goods will shoot up and they will become less competitive in the international market at the same exchange rate. So, in this case, the RBI can reduce the exchange rate of a currency.

This will cause a devaluation of domestic currency as compare to another foreign currency. This will again make domestic goods more competitive in the international market.

28. Which of the following investors/ agencies can purchase the government of India securities/bonds?

1. Reserve Bank of India

2. Portfolio Investors

3. Financial Institutions

Select the correct answer using the code is given below:

A. 1 only

B. 1 and 3 only

C. 3 only

D. All of the above

Answer: D

Explanation: All the participants given above can buy and sell government securities.

29. Which of the following statements will be true if the inflation in the economy is increasing?

1. The bond price will decrease

2. Bondholders will loose

3. The yield on bonds will increase

Select the correct answer using the code given below:

A. 1 only

B. 1 and 2 only

C. 3 only

D. All of the above

Answer: D

Explanation:

A bond (debt paper) holder is expected to get a fixed interest regularly and principal at maturity. But if the inflation in the economy starts increasing the price of the bond decreases (because now the actual value of the principal and interest which the bondholder will get will be of less value) and bondholders lose.

When the price of the bond decreases in the market, the person who will purchase the bond will have to pay less price Government Securities (G-Sec) and hence he will get more return/yield. (The interest rate on the bond remains fixed but its price

Page 35: Free . Regular . Quality€¦ · 1. Currency notes are legal tenders 2. Currency notes are unlimited legal tenders 3. Currency notes are guaranteed by the Central Government 4. Currency

www.YouTube.com/SleepyClasses

www.sleepyclasses.com/

fluctuates in the market and hence the return also fluctuates.

If the market price of the bond is low, then the return/yield on the bond will be high. This is because the person who will purchase the bond will have to pay fewer prices to get the same bond).

30. Consider the following statements:

1. Currency notes are legal tenders

2. Currency notes are unlimited legal tenders

3. Currency notes are guaranteed by the Central Government

4. Currency notes are guaranteed by the RBI

Select the correct answer using the code given below:

A. 1 and 3 only

B. 1 and 4 only

C. 1, 2 and 3 only

D. 2 and 4 only

Answer: C

Explanation:

A country or its citizens may use many modes of exchange in their daily lives. ‘Legal tender’ is the money that is recognized by the law of the land, as valid for payment of a debt. It must be accepted for the discharge of debt. RBI Act 1934, Section 26 states that “Every central bank note shall be legal tender at any place in India in payment or on account for the amount expressed therein”.

Legal tender can be limited or unlimited in character. In India, coins function as limited legal tender. Therefore, 50 paise coins can be offered as legal tender for dues

up to Rs.10 and smaller coins for dues up to Rs.1. Currency notes are unlimited legal tender and can be offered as payment for dues of any size. As per the RBI Act 1934, all currency notes are guaranteed by the Central Government

31. Consider the following statements regarding the insurance cover

provided to depositors by Deposit Insurance and Credit Guarantee Corporation (DICGC)

1. All commercial banks and urban cooperative banks have to register with DICGC for providing insurance to depositors

2. RBI incurs the insurance premium burden

3. Government incurs the insurance

premium burden

Select the incorrect answer using the code is given below:

A. 1 only

B. 1 and 2 only

C. 3 only

D. 2 and 3 only

Answer: D

Explanation:

As per "The Deposit Insurance and Credit Guarantee Corporation (DICGC) Act 1961", DICGC must register all commercial banks (scheduled and non-scheduled both) and Urban Cooperative Banks (UCB) and State and District Central Cooperative Banks (StCB/DCCB) as an insured bank.

(StCB/DCCB are rural cooperative banks) And every insured bank is liable to pay a premium to DICGC as

Page 36: Free . Regular . Quality€¦ · 1. Currency notes are legal tenders 2. Currency notes are unlimited legal tenders 3. Currency notes are guaranteed by the Central Government 4. Currency

www.YouTube.com/SleepyClasses

www.sleepyclasses.com/

may be notified by DICGC after the approval of RBI. But the premium shall not exceed fifteen paise per annum for every hundred rupees of the total amount of the deposits in that bank. Which means premium has been capped under the DICGC Act. As per the rules, the premium cost is required to be borne by the bank themselves and cannot be passed on to depositors.

Since the insurance cover has been increased from the present Rs. 1 lakh per depositor per bank to Rs. 5 lakh per depositor per bank, the insurance premium has also been increased from presently 10 paise per Rs. 100 of deposit to 12 paise per Rs. 100 deposit. Deposits of foreign governments, deposits of central and state governments, and interbank deposits are not covered/insured.

32. Which of the following were the reasons for the recent NBFC crisis in the economy?

1. Relying on short term financing

to fund long-term investments

2. Asset liability mismatch (ALM)

3. Rollover risk of commercial papers

Select the correct answer using the code given below:

A. 1 and 3 only

B. 1 and 2 only

C. 2 and 3 only

D. All of the above

Answer: D

Explanation:

NBFCs rely on short-term financing like commercial papers to fund long-term investments (long term loans to businesses). So, the tenure of liability

(the commercial papers issued by NBFCs) is short and the tenure of the asset (loans are given by NBFCs) is long.This is called Asset Liability Mismatch (ALM). So, NBFCs are required to refinance these commercial papers at short frequencies of a few months.

The frequent repricing of loans/advances (as they need to be raised again and again and interest rate keeps on changing in the market) exposes NBFCs to the risk of facing higher financing costs, and in the worst case, credit rationing. Such refinancing risks are referred to as rollover risks.

33. There is a Human Development Report (HDR) released by UNDP in December 2019. It talks about various kinds of inequalities. Consider the following statements in this regard

1. Climate change and technological transformation have the potential to deepen existing social and economic fault lines.

2. More men are free from gender bias as compared to a woman.

Which of the following statements are NOT true?

A. 1 only

B. 2 only

C. Both 1 and 2

D. None of the above

Answer: B

Explanation:

There is a human development index (HDI) report by the United Nations Development Programme (UNDP).

The report presents a new index indicating how prejudices and social

Page 37: Free . Regular . Quality€¦ · 1. Currency notes are legal tenders 2. Currency notes are unlimited legal tenders 3. Currency notes are guaranteed by the Central Government 4. Currency

www.YouTube.com/SleepyClasses

www.sleepyclasses.com/

beliefs obstruct gender equality, which shows that only 14% of women and 10% of men worldwide have no gender bias.

The report notes that this indicates a backlash to women’s empowerment as these biases have shown growth especially in areas where more power is involved, including in India. The report also highlights that new forms of inequalities will manifest in the future through climate change and technological transformation which have the potential to deepen existing social and economic fault lines.

34. Consider the following statements regarding the National Pension System

1. NPS has fixed interest rates decided by the Ministry of Finance.

2. A unique Permanent Retirement Account Number (PRAN) is generated and maintained by the Central Recordkeeping Agency (CRA) for individual subscribers.

3. NPS also has the facility of Tier-II account which is a voluntary account, which offers liquidity of investments and withdrawals.

4. On exit/retirement/superannuation, a minimum of 85% of the corpus is mandatorily utilized to procure a pension for life by purchasing an annuity from a life insurance company.

Which of the following statements are true?

A. 1 and 2 only

B. 2 and 3 only

C. 3 and 4 only

D. 1, 3 and 4 only

Answer: B

Explanation:

The National Pension System (NPS) is being administered and regulated by Pension Fund Regulatory and Development Authority (PFRDA) set up under the PFRDA Act, 2013.

NPS is a market-linked, defined contribution product. Under NPS, a unique Permanent Retirement Account Number (PRAN) is generated and maintained by the Central Recordkeeping Agency (CRA) for an individual subscriber.

NPS offers two types of accounts, namely Tier-I and Tier-II. Tier-I account is the pension account having restricted withdrawals. Tier-II is a

voluntary account, which offers liquidity of investments and withdrawals. It is allowed only when there is an active Tier-I account in the name of the subscriber.

The contributions accumulate over a period of time until retirement grows with market-linked returns. On exit/retirement/superannuation, a minimum of 40% of the corpus is mandatorily utilized to procure a pension for life by purchasing an annuity from a life insurance company and the balance corpus is paid as a lump sum.

35. With reference to ‘voting in India’ which of the following statements is/are correct:

1. In India, Voting can be done in three ways: in person, through posts, and through a proxy.

2. Voting through post and proxy is only allowed to NRIs.

3. Lok Sabha recently passed

Representation of the People (Amendment) Bill, 2017 to provide for an online voting facility to NRIs

Page 38: Free . Regular . Quality€¦ · 1. Currency notes are legal tenders 2. Currency notes are unlimited legal tenders 3. Currency notes are guaranteed by the Central Government 4. Currency

www.YouTube.com/SleepyClasses

www.sleepyclasses.com/

Select the correct answer using the codes given below:

A. 1 only

B. 1 and 2 only

C. 1 and 3 only

D. 1,2 and 3

Answer: A

Explanation:

STATEMENT 1 is correct: In India, voting can be done in three ways: in person, through the post, and through a proxy.

STATEMENT 2 is incorrect: Voting through post and proxy is only allowed to a “classified service voter” — a definition that includes members of the armed forces, BSF, CRPF, CISF, General Engineering Reserve Force and Border Road Organisation.

STATEMENT 3 is incorrect: Lok Sabha recently passed Representation of the People (Amendment) Bill, 2017 to provide for a proxy voting facility to NRIs.

36. Consider the following statements about ‘Election Commissioners of India’:

1. The appointment of the chief election commissioner and other election commissioners shall be made by the president.

2. The election commissioners have equal powers and receive equal salary, allowances, and other perquisites, which are similar to those of a judge of the Supreme Court.

Which of the statements given above is/are correct?

A. 1 only

B. 2 only

C. Both 1 and 2

D. Neither 1 nor 2

Answer: C

Explanation:

STATEMENT 1 is correct : The appointment of the chief election commissioner and other election commissioners shall be made by the president.

STATEMENT 2 is correct : The election commissioners have equal powers and receive equal salary, allowances and other perquisites, which are similar to those of a judge of the Supreme Court.

37. Consider the following statements:

1. The Legislative Council of a State in India can be larger in size than half of the Legislative Assembly of that particular State.

2. The Governor of a State nominates the Chairman of the Legislative Council of that particular State.

3. The State Assembly can abolish a legislative council or create it on its own.

Which of the statements given above is/are incorrect?

A. 1 only

B. 1 and 3 only

C. 1 and 2 only

D. 1, 2 and 3

Answer: D

Explanation:

STATEMENT 1 is incorrect: The maximum strength of the State Council is fixed at one-third of the total strength of the assembly and the minimum strength is fixed at 40. It

Page 39: Free . Regular . Quality€¦ · 1. Currency notes are legal tenders 2. Currency notes are unlimited legal tenders 3. Currency notes are guaranteed by the Central Government 4. Currency

www.YouTube.com/SleepyClasses

www.sleepyclasses.com/

means that the size of the council depends on the size of the assembly of the concerned state.

STATEMENT 2 is incorrect: The Chairman of the Legislative Council is elected by the council itself from amongst its members and not by the governor.

STATEMENT 3 is incorrect: The Parliament can abolish a legislative council (where it already exists) or create it (where it does not exist), if the legislative assembly of the concerned state passes a resolution to that effect.

38. Which of the following Directive Principle was not added by the 42nd

Amendment Act of 1976?

A. To secure opportunities for the healthy development of children (Article 39).

B. To promote equal justice and to provide free legal aid to the poor (Article 39 A).

C. To take steps to secure the participation of workers in the management of industries (Article 43 A).

D. State to minimize inequalities in income, status, facilities, and

opportunities (Article 38)

Answer: D

Explanation:

The 42nd Amendment Act of 1976 added four new Directive Principles to the original list. They require the State:

To secure opportunities for the healthy development of children (Article 39)

To promote equal justice and to

provide free legal aid to the poor (Article 39 A).

To take steps to secure the participation of workers in the management of industries (Article 43 A).

To protect and improve the environment and to safeguard forests and wildlife (Article 48 A).

The 44th Amendment Act of 1978 added one more Directive Principle, which requires the State to minimize

39. Consider the following statements about ‘National Commission for Scheduled Castes’:

1. It is a constitutional body.

2. NCSC has the power to look into complaints and welfare measures with regard to Scheduled Castes, backward classes, and Anglo-Indians.

3. The commission presents an annual report to the Parliament.

Which of the statements given above is/are correct?

A. 1 only

B. 1 and 2 only

C. 1 and 3 only

D. 2 and 3 only

Answer: B

Explanation:

STATEMENT 1 is correct: The National Commission for Scheduled Castes (SCs) is a constitutional body in the sense that it is directly established by Article 338 of the Constitution. It consists of a chairperson, a vice-chairperson and three other members. They are appointed by the President by warrant under his hand and seal. Their conditions of service and tenure of office are also determined by the President.

Page 40: Free . Regular . Quality€¦ · 1. Currency notes are legal tenders 2. Currency notes are unlimited legal tenders 3. Currency notes are guaranteed by the Central Government 4. Currency

www.YouTube.com/SleepyClasses

www.sleepyclasses.com/

STATEMENT 2 is incorrect: the 102 and constitutional amendment act the NCBC has been given the mandate of looking into the complaints for backward classes and not NCSC

STATEMENT 3 is incorrect: The commission presents an annual report to the president.

40. Which of the following is/are a

constitutional body?

1. National Human Rights Commission.

2. Central Information Commission

3. Union Public Service Commission

4. Finance Commission

Choose using the correct code:

A. 1 and 3 only

B. 1, 2 and 3 only

C. 3 only

D. 3 and 4 only

Answer: D

Explanation:

STATEMENT 1: NHRC is a statutory body. It is not a constitutional body

STATEMENT 2: CIC is a statutory body.

STATEMENT 3: UPSC is a constitutional body.

STATEMENT 4: Finance commission is a constitutional body.

41. Consider the following statements regarding the Parliamentary Standing Committee on Finance:

1. The Chairman of the committee is nominated by the chairman of Rajya Sabha.

2. It consists of 31 members, 21 from Lok Sabha and 10 from Rajya Sabha.

Which of the above statement is correct?

A. 1 only

B. 2 only

C. Both 1 and 2

D. Neither 1 nor 2

Answer: B

Explanation:

STATEMENT 1 is incorrect: As Chairman of the committee is nominated by the speaker of Lok Sabha and not by the chairman of Rajya Sabha. Parliamentary Standing Committee on Finance comes under Lok Sabha and not under Rajya Sabha.

STATEMENT 2 is correct: As standing committee consists of 31 members, 21 members from Lok Sabha and 10 from Rajya Sabha.

42. Which committee of parliament considers and advises on matters concerning the affairs of the house, which do not fall within the jurisdiction of any other parliamentary committee.

A. General Purpose Committee

B. Rules Committee of Lok Sabha

C. Business Advisory Committee

D. Committee on Subordinate legislation

Answer: A

Explanation:

General Purpose committee considers and advises on matters concerning the affairs of the house, which do not fall within the jurisdiction of any other parliamentary committee.

Page 41: Free . Regular . Quality€¦ · 1. Currency notes are legal tenders 2. Currency notes are unlimited legal tenders 3. Currency notes are guaranteed by the Central Government 4. Currency

www.YouTube.com/SleepyClasses

www.sleepyclasses.com/

Rules Committee of Lok Sabha‘s functions is to consider the matters of procedure and conduct of business in the house and recommends and necessary amendments

Business Advisory Committee regulates the programme and time table of the house.

Committee on Subordinate legislation examines and reports to the house whether the powers to make regulations, rules, sub-rules and bye-laws delegated by the Parliament.

43. Consider the following statements:

1. Indian Constitution does not have any provision for categorization of any state as a Special Category Status State.

2. For special category status category states, the Centre pays 90 percent of the funds required in a centrally-sponsored scheme as against 60 percent in case of normal category states.

3. Low population density or the presence of a sizeable tribal population is one of the features required to get Special Category Status.

Which of the statements given above is/are correct?

A. 1 only

B. 1 and 3 only

C. 2 only

D. 1, 2 and 3

Answer: D

Explanation:

The Constitution does not include any provision for the categorization of any State in India as a Special Category Status (SCS) State. But, recognizing that some regions in the country were historically disadvantaged in contrast

to others, Central plan assistance to the SCS States has been granted in the past by the erstwhile Planning Commission body, National Development Council (NDC). The NDC granted this status based on a number of features of the States which included:

hilly and difficult terrain

low population density or the presence of sizeable tribal population

strategic location along international borders

economic and infrastructural backwardness

Non-viable nature of State finances.

For special category status category states, the Centre pays 90 percent of the funds required in a centrally-sponsored scheme as against 60 percent in the case of normal category states, while the remaining funds are provided by the state governments. the special category state status was effectively removed as per the recommendations of the 14th Finance

Commission and the formation of NITI Aayog

44. With reference to the provisions of ‘no-

confidence motion’ consider the following statements:

1. The motion needs the support of one-third members to be admitted to the lower house.

2. It need not state the reasons for its adoption in the Lok Sabha.

3. It can be moved against an individual minister or a group of ministers or the entire council of

ministers.

Which of the statements given above is/are incorrect?

Page 42: Free . Regular . Quality€¦ · 1. Currency notes are legal tenders 2. Currency notes are unlimited legal tenders 3. Currency notes are guaranteed by the Central Government 4. Currency

www.YouTube.com/SleepyClasses

www.sleepyclasses.com/

A. 1 only

B. 1 and 3 only

C. 2 only

D. 1, 2 and 3

Answer: B

Explanation:

A no-confidence motion can be moved by any member of the House. It can be moved only in the Lok Sabha and not Rajya Sabha. Rule 198 of the Rules of Procedure and conduct of Lok Sabha specifies the procedure for moving a no-confidence motion. The member has to give written notice of the motion before 10 am which will be read out by the Speaker in the House.

A minimum of 50 members have to accept the motion and accordingly, the Speaker will announce the date for discussion for the motion. The allotted date has to be within 10 days from the day the motion is accepted. Otherwise, the motion fails and the member who moved the motion will be informed about it. Unlike Censure motion, it need not state the reasons for its adoption in the Lok Sabha.

45. ‘NOTA’ option is available to voters

in which of the following elections:

1. Lok Sabha elections

2. Rajya Sabha elections

3. State Assembly elections

4. State Council elections

Select the correct answer using the codes given below:

A. 1 and 3 only

B. 2 and 4 only

C. 1 only

D. 1, 2, 3, and 4

Answer: A

Explanation:

NOTA: ''NOTA'' or ''none of the above''

is a ballot option provided in the elections to Indian voters. Through NOTA, a citizen has the right to not vote for any candidate contesting the elections.

NOTA is an option is direct elections like Lok Sabha and State Assemblies. Whereas elections to Rajya Sabha and State Councils are Indirect elections

46. With reference to the constitutional provisions for the minority in India consider the following statement:

1. The Constitution of India does not define the word minority.

2. Gujarat became the first state in India to grant minority status for Jews.

3. Muslims, Christians, Sikhs, Zoroastrians, Buddhists, and Jains are the only communities to which the Union Government has accorded the status of the minority.

Which of the statements given above is/are correct?

A. 1 only

B. 1 and 3 only

C. 3 only

D. 1, 2 and 3

Answer: B

Explanation:

STATEMENT 2 is incorrect: Gujarat becomes the third state in India after Maharashtra and West Bengal to have granted minority status for Jews.

Page 43: Free . Regular . Quality€¦ · 1. Currency notes are legal tenders 2. Currency notes are unlimited legal tenders 3. Currency notes are guaranteed by the Central Government 4. Currency

www.YouTube.com/SleepyClasses

www.sleepyclasses.com/

The Constitution of India does not define the word minority but refers to the word while according rights to religious and linguistic minorities and enshrines various provisions like article 25, 29, 30 for the protection of the rights and interests of the minorities.

The Union Government accords the status of minority communities to six religious communities viz. Muslims, Christians, Sikhs, Zoroastrians (Parsis), Buddhists, and Jains.

47. With reference to ‘North Eastern

Council (NEC)’, consider the following statements

1. It is a statutory body with Governors and Chief Ministers of all the eight North Eastern States as its Member.

2. Minister of DoNER (Development of North Eastern Region) is ex-officio Chairman of the North Eastern Council (NEC).

Which of the statements given above is/are correct?

A. 1 only

B. 2 only

C. Both 1 and 2

D. Neither 1 nor 2

Answer: A

Explanation:

STATEMENT 1 is correct: as North Eastern Council (NEC) is a statutory body with Governors and Chief Ministers of all the eight North Eastern States as its Member. NEC implements various projects through the State and Central agencies. NEC was established under the North Eastern Council Act, 1971 as an apex level body for securing balanced and

coordinated development and facilitating coordination with the States. Subsequent to the Amendment of 2002, NEC has been mandated to function as a regional planning body for the North Eastern Area and while formulating a regional plan for this area, shall give priority to the schemes and projects benefiting two or more states provided that in the case of Sikkim, the Council shall formulate specific projects and schemes for that State.

STATEMENT 2 is incorrect: as Minister of DoNER (Development of North Eastern Region) is not ex-officio Chairman of North Eastern Council (NEC). Under the new arrangement with Home Minister as Chairman and Minister of DoNER as Vice Chairman, NEC and all the Governors and Chief Ministers of North Eastern States

as Member would provide a forum for discussing inter-state matters more comprehensively and also consider common approaches to be taken in future

48. With reference to ‘The Law Commission of India’, consider the following statements:

1. The Law Commission of India is a statutory body constituted through the act of Parliament.

2. It works towards the progressive development and codification of the laws of the country.

Which of the statements given above is/are correct?

A. 1 only

B. 2 only

C. Both 1 and 2

D. Neither 1 nor 2

Page 44: Free . Regular . Quality€¦ · 1. Currency notes are legal tenders 2. Currency notes are unlimited legal tenders 3. Currency notes are guaranteed by the Central Government 4. Currency

www.YouTube.com/SleepyClasses

www.sleepyclasses.com/

Answer: B

Explanation:

STATEMENT 1 is incorrect as The Law Commission of India is a non-statutory body constituted by the Government of India from time to time. The Commission was originally constituted in 1955 and is reconstituted every three years. It was not established through an Act of Parliament

STATEMENT 2 is correct as Law Commission of India works towards the progressive development and codification of the laws of the country. Its major function is to work for legal reform. Its membership primarily comprises legal experts, who are entrusted a mandate by the Government. The Commission is established for a fixed tenure and works as an advisory body to the Ministry of Law and Justice

49. Consider the following statements about Deputy Chairman of Rajya Sabha:

1. In the election of Deputy Chairman of Rajya Sabha, members of both Lok Sabha and Rajya Sabha participate.

2. Deputy Chairman of Rajya Sabha is subordinate to Chairman of Rajya Sabha and is not directly responsible to Rajya Sabha.

Which of the above statement is Correct?

A. 1 only

B. 2 only

C. Both 1 and 2

D. Neither 1 nor 2

Answer: D

Explanation:

STATEMENT 1 is incorrect as in the election of Deputy Chairman of Rajya

Sabha only member of Rajya Sabha participate and not a member of Lok Sabha.

STATEMENT 2 is also incorrect as Deputy Chairman of Rajya Sabha is not subordinate to Chairman of Rajya Sabha and is directly responsible to Rajya Sabha.

50. Consider the following statements:

1. A Supreme Court judge holds the office till the age of 62.

2. SC judge can only be removed on the ground of ‘misbehaviour’

3. SC judge is appointed by the President on the sole advice from Chief Justice of India.

Which of the statements given above is/are correct?

A. 1 and 2 only

B. 2 and 3 only

C. 2 only

D. None

Answer: D

Explanation:

STATEMENT 1 is incorrect: A Supreme

Court judge holds the office till the age of 65 whereas an HC judge holds the office till the age of 62. STATEMENT 2 is incorrect: The grounds of removal are two—proved misbehavior or incapacity.

STATEMENT 3 is incorrect: The judge of SC is appointed by President on Advice of CJI who has to first consult with the SC collegium before forwarding the name and the name finalized by collegium would be final.

Page 45: Free . Regular . Quality€¦ · 1. Currency notes are legal tenders 2. Currency notes are unlimited legal tenders 3. Currency notes are guaranteed by the Central Government 4. Currency

www.YouTube.com/SleepyClasses

www.sleepyclasses.com/

51. Consider the following statement about the seismic wave:

1. P waves create crests and troughs while travelling through the earth.

2. S waves create compressions and expansions while travelling through earth.

3. S waves reach to the surface earlier than P waves.

Choose the correct answer from the following code:

A. 1 and 2 Only

B. 2 and 3 Only

C. All of the above

D. None of the above

Answer: D

Explanation:

52. Out of the following which one is not an

Intrusive volcanic landform?

A. Lacoliths

B. Sills

C. Cinder cone

D. Dyke

Answer: C

Explanation:

53. Out of the following, which are formed

by erosion

1. River terrace

2. Incised Meander

3. Fans

4. Deltas

Choose the correct answer from the following code:

A. 1 and 2 Only

B. 1,2 and 4 Only

C. 2,3 and 4 Only

D. 2 and 3 Only

Answer: A

Explanation:

River terraces are surfaces marking old valley floor or floodplain levels. They may be bedrock surfaces without any alluvial cover or alluvial terraces consisting of stream deposits. River terraces are basically products of erosion as they result due to

Page 46: Free . Regular . Quality€¦ · 1. Currency notes are legal tenders 2. Currency notes are unlimited legal tenders 3. Currency notes are guaranteed by the Central Government 4. Currency

www.YouTube.com/SleepyClasses

www.sleepyclasses.com/

vertical erosion by the stream into its own depositional floodplain.

It is common to find meandering courses over floodplains and delta plains where stream gradients are very gentle. But very deep and wide meanders can also be found cut in hard rocks. Such meanders are called incised or entrenched meanders

54. Which of the given statement/statements is/are correct?

1. Metamorphic rock is a result of a transformation of a pre-existing rock.

2. Uplift and erosion are the two forces behind the formation of metamorphic rocks.

Choose the correct answer from the following Code:

A. 2 Only

B. None of the above

C. Both 1 and 2

D. 1 only

Answer: D

Explanation: Metamorphic rocks are formed due to the transformation of a pre-existing rock.

They are predominantly sedimentary or igneous rocks that have undergone physical and chemical changes under the action of extreme heat and pressure. Formation of metamorphic rocks can take place under different physical conditions i.e. in different temperatures (up to 200 C) and pressures (up to 1500 bars).

Their natural characteristics undergo a massive transformation in such extreme conditions of temperature and pressure. Metamorphic rocks make up a large part of the Earth's crust and

form 12% of the Earth's land surface. Gneissoid, granite, syenite, slate, schist, marble, quartzite, etc. are some examples of metamorphic rocks.

55. The temperature of a place is not affected

by:

A. Air Mass

B. Ocean currents

C. Longitude

D. Altitude

Answer: C

Explanation: Latitude affects the temperature

distribution of a place and not its longitude as latitude determines the insolation received in an area and thereby its temperature.

56. Which of the following is correct about the Laterite soils?

1. These soils develop in the area of low temperature and low rainfall.

2. These soils are rich in humus content.

3. These soils are highly infertile and poorly cultivated.

Which of the statements given above is/are correct?

A. 3 only

B. 2 and 3 only

C. 1 and 3 only

D. All of the above

Answer: A

Explanation:

Laterite has been derived from the Latin word ‘Later’ which means brick. The laterite soils develop in areas with high temperatures and high rainfall. These are the result of intense

Page 47: Free . Regular . Quality€¦ · 1. Currency notes are legal tenders 2. Currency notes are unlimited legal tenders 3. Currency notes are guaranteed by the Central Government 4. Currency

www.YouTube.com/SleepyClasses

www.sleepyclasses.com/

leaching due to tropical rains. With rain, lime and silica are leached away, and soils rich in iron oxide and aluminum compound are left behind.

Humus content of the soil is removed fast by bacteria that thrive well at high temperatures. These soils are poor in organic matter, nitrogen, phosphate, and calcium, while iron oxide and potash are in excess. Hence, laterites are not suitable for cultivation; however, the application of manures and fertilizers are required for making the soil fertile for cultivation.

Red laterite soils in Tamil Nadu, Andhra Pradesh, and Kerala are more suitable for tree crops like cashew nuts.

Laterite soils are widely cut as bricks for use in house construction. These soils have mainly developed in the higher areas of the peninsular plateau.

57. Which of the following is not correct

about the air mass?

1. The air with distinctive characteristics in terms of temperature and humidity is called an air mass.

2. Air mass has little vertical variation in temperature and moisture.

3. The interaction of cold and warm air mass leads to rain.

Choose from the following code:

A. 1 Only

B. 2 only

C. 2 and 3 Only

D. All the above

Answer: B

Explanation:

When the air remains over a homogenous area for a sufficiently

longer time, it acquires the characteristics of the area. The homogenous regions can be the vast ocean surface or vast plains. The air with distinctive characteristics in terms of temperature and humidity is called an airmass.

It is defined as a large body of air having little horizontal variation in temperature and moisture. The homogenous surfaces, over which air masses form, are called the source regions. When two different air masses meet, the boundary zone between them is called a front. The process of formation of the fronts is known as frontogenesis.

58. Consider the statements:

1. Continental shelves contain lesser fossil fuel as compare to continental shelves.

2. Canyons and trenches are observed in the continental shelves

Choose the incorrect answer from the following code:

A. 1 Only

B. Both 1 and 2

C. 2 only

D. None of the above

Answer: B

Explanation:

When the air remains over a homogenous area for a sufficiently longer time, it acquires the characteristics of the area. The homogenous regions can be the vast ocean surface or vast plains. The air with distinctive characteristics in terms of temperature and humidity is called an airmass.

Page 48: Free . Regular . Quality€¦ · 1. Currency notes are legal tenders 2. Currency notes are unlimited legal tenders 3. Currency notes are guaranteed by the Central Government 4. Currency

www.YouTube.com/SleepyClasses

www.sleepyclasses.com/

It is defined as a large body of air having little horizontal variation in temperature and moisture. The homogenous surfaces, over which air masses form, are called the source regions. When two different air masses meet, the boundary zone between them is called a front. The process of formation of the fronts is known as frontogenesis.

59. The ocean water temperature is affected by

1. Distribution of land and water

2. Prevailing wind

3. Ocean current

Choose the answer from the following code:

A. 2 and 3 Only

B. 1 and 3 Only

C. 1 and 2 Only

D. All the above

Answer: D

Explanation:

Unequal distribution of land and

water: the oceans in the northern hemisphere receive more heat due to their contact with a larger extent of land than the oceans in the southern hemisphere.

Prevailing wind: the winds blowing from the land towards the oceans drive warm surface water away from the coast resulting in the upwelling of cold water from below. It results in the longitudinal variation in the temperature. Contrary to this, the onshore winds pile up warm water near the coast and this raises the temperature.

Ocean currents: warm ocean currents raise the temperature in cold areas

while the cold currents decrease the temperature in warm ocean areas. Gulf stream (warm current) raises the temperature near the eastern coast of North America and the West Coast of Europe while the Labrador current (cold current) lowers the temperature near the northeast coast of North America.

60. Consider the following statements about soil erosion.

1. Gully erosion takes place on level lands after a heavy shower and the soil removal is not easily noticeable.

2. Sheet erosion is common on steep slopes.

3. A region with a large number of ravines is called badland topography.

Which of the given statements is/are correct?

A. 2 Only

B. 1 and 2 only

C. 2 and 3 only

D. None of the above

Answer: C

Explanation:

Sheet erosion takes place on level lands after a heavy shower and the soil removal is not easily noticeable. But it is harmful since it removes the finer and more fertile top soil. Gully erosion is common on steep slopes. Gullies deepen with rainfall, cut the agricultural lands into small fragments and make them unfit for cultivation.

The Badlands are a type of arid terrain with clay-rich soil that has been extensively eroded by wind and water. Canyons, ravines, gullies, hoodoos and

Page 49: Free . Regular . Quality€¦ · 1. Currency notes are legal tenders 2. Currency notes are unlimited legal tenders 3. Currency notes are guaranteed by the Central Government 4. Currency

www.YouTube.com/SleepyClasses

www.sleepyclasses.com/

other such geological forms are common in The Badlands.

61. The increasing order of salinity among the following water bodies is

1. Dead sea

2. Red sea

3. Atlantic Ocean

4. Black Sea

Choose the correct answer from the following code:

A. 4<3<2<1

B. 3<4<1<2

C. 4<3<2<1

D. 3<2<4<1

Answer: C

Explanation:

62. Consider the following statement about sugarcane crop:

1. It requires a hot and humid climate.

2. Too heavy rainfall results in high sugar content.

3. Short cool dry winter season during ripening and harvesting is ideal.

4. South India offers favourable climatic conditions for the growth of sugarcane.

Choose the correct statements

A. 1, 2 and 4 only

B. 2, 3 and 4 only

C. 1, 3 and 4 only

D. All of the above

Answer: C

Explanation: Dihang It is a long duration crop and requires 10 to 15 and even 18 months to mature, depending upon the geographical conditions. It requires a hot and humid climate with an average temperature of 21°-27°C and 75-150 cm rainfall.

In the latter half, a temperature above 20°C combined with an open sky helps in acquiring juice and its thickening. Too heavy rainfall results in low sugar content and deficiency in rainfall produces fibrous crops. Irrigation is required in areas receiving lesser rainfall than the prescribed limit. Short cool dry winter season during ripening and harvesting is ideal.

Frost is detrimental to sugarcane. Therefore, it must be harvested before frost season, if it is grown in northern parts of the country where winters are very cold and frost is a common phenomenon. On the other hand, hot dry winds are also inimical to sugarcane.

It can grow on a variety of soils including loams, clayey loams, black cotton soils, brown or reddish loams, and even laterites. In fact, sugarcane can tolerate any kind of soil that can retain moisture. But deep rich loamy soils are ideal for its growth.

63. The Black soils lack in:

1. Phosphorous

2. Nitrogen

3. Lime

4. Organic matter

Select the correct answer from the code given below:

A. 1, 3 and 4 only

Page 50: Free . Regular . Quality€¦ · 1. Currency notes are legal tenders 2. Currency notes are unlimited legal tenders 3. Currency notes are guaranteed by the Central Government 4. Currency

www.YouTube.com/SleepyClasses

www.sleepyclasses.com/

B. 1 and 4 only

C. 1, 2, 3 and 4 only

D. 1, 2, and 4 only

Answer: D

Explanation:

Black soil is also known as cotton soil. This is the third largest group in India. This soil is formed from rocks of cretaceous lava. This stretch over the parts of Gujarat, Maharashtra, Western parts of Madhya Pradesh, North- Western Andhra Pradesh, Karnataka, Tamil Nadu, Rajasthan, Chhattisgarh, Jharkhand up to Raj Mahal hills.

The soil is rich in iron, lime, calcium,

potash, magnesium, and aluminum. It has high water retaining capacity and good for cotton cultivation, Tobacco, citrus fruits, castor, and linseed

64. Indian states/UTs which are the part of Cauvery basin are-

1. Tamil Nadu

2. Kerala

3. Karnataka

4. Goa

Select the correct answer from the code given below:

A. 1, 2 and 4 only

B. 1, 2 and 3 only

C. 2, 3 and 4 only

D. 1, 2, 3 and 4

Answer: B

Explanation: River Cauvery rises in southwestern Karnataka, it flows southeast some 800 kilometers to enter the Bay of Bengal. River's basin covers three states –

Tamil Nadu, Kerala and Karnataka - and a Union Territory – Puducherry.

65. Consider the following state Forestry in India.

1. Agro-forestry is the raising of trees and agriculture crops on the same land inclusive of the waste patches.

2. Farm forestry is a process under which farmers grow trees for commercial non-commercial purposes on lands.

3. Social forestry means afforestation of barren lands with the purpose of helping in the environmental, social and rural development

Which of the given statements is/correct?

A. 3 only

B. 1 and 2 only

C. 1, 2 and 3

D. None of the above

Answer: C

Explanation:

Agroforestry focuses on the wide range of trees grown on farms and other rural areas. Agroforestry is a collective name for land-use systems involving trees combined with crops and/or animals on the same unit of land.

Farm Forestry: Farm forestry is the name given to programmes which promote commercial tree growing by farmers on their own land. Farm forestry was defined as the practice of forestry in all its aspects in and around the farms or village lands integrated with other farm operations.

Social Forestry means the management and protection of forests as well as afforestation of barren lands with the purpose of helping in the

Page 51: Free . Regular . Quality€¦ · 1. Currency notes are legal tenders 2. Currency notes are unlimited legal tenders 3. Currency notes are guaranteed by the Central Government 4. Currency

www.YouTube.com/SleepyClasses

www.sleepyclasses.com/

environmental, social and rural development. Social forestry is forestry for community development. Thus, it is people-oriented, value-based management of forests with a major objective of satisfying the needs, wants and aspirations of both the people and the government.

66. The break-in ongoing monsoon is mainly caused by-

1. In the northern region, a break occurs due to the absence of rain-bearing storm in the monsoon trough.

2. When winds run parallel to the western coast

Choose the correct answer from the following code:

A. 1 Only

B. 2 Only

C. Both 1 and 2

D. None of the above

Answer: C

Explanation:

During the south-west monsoon period after having rained for a few days, if rain fails to occur for one or more weeks, it is known as a break in the monsoon. These dry spells are quite common during the rainy season.

These breaks in the different regions are due to different reasons:

In northern India, rains are likely to fail if the rain-bearing storms are not very frequent along the monsoon trough or the ITCZ over this region.

Over the west coast, the dry spells are associated with days when winds blow parallel to the coast.

67. Consider the following statements with respect to RAIDER – X

1. It has been developed by DRDO.

2. It is an explosive bomb.

Which of the above is/are correct?

A. 1 only

B. 2 only

C. Both

D. None

Answer: A

Explanation:

It is an explosion detection device.

Also, The Defence Research and Development Organisation (DRDO) and the Indian Institute of Science in Bangalore have developed a new bomb detection device called Raider-X. It can detect up to 20 explosives from a stand-off distance of two meters. Developed by the High Energy Material Research Laboratory (HEMRL) — an arm of the DRDO — in Pune and the Indian Institute of Science in Bangalore.

It can also discern bulk explosives even if they have been concealed.

68. Which of the following points with respect to Coal Bed Methane (CBM) are correct

1. It is a conventional form of natural gas found in coal deposits and coal seams.

2. CBM is formed during the process of coalification, the transformation of plant material into coal.

Choose the correct statement

A. 1 only

B. 2 only

C. Both

Page 52: Free . Regular . Quality€¦ · 1. Currency notes are legal tenders 2. Currency notes are unlimited legal tenders 3. Currency notes are guaranteed by the Central Government 4. Currency

www.YouTube.com/SleepyClasses

www.sleepyclasses.com/

D. None

Answer: B

Explanation: Coalbed methane (CBM) is an unconventional form of natural gas found in coal deposits or coal seams. CMB is formed during the process of coalification, the transformation of plant material into coal.

69. Which of the following statement(s)

is/are correct with respect to ExoMArs

1. It is a joint endeavor of ESA and Russian space agency, Roscosmos.

2. Its goal is to address the question of whether life has ever existed on Mars.

Choose the correct statement

A. 1 only

B. 2 only

C. Both

D. None

Answer: C

Explanation:

It is an astrobiology programme by the ESA and the Russian space agency Roscosmos. The goals of ExoMars are to search for signs of past life on Mars investigate how the Martian water and geochemical environment varies, investigate atmospheric trace gases and their sources and by doing so demonstrate the technologies for a future Mars sample-return mission

The launch of the ExoMars rover has been delayed to 2022.

70. Currently, India’s National Supercomputing Mission was in news, consider the following statements regarding the same

1. It is being implemented by department of science and technology (DST) and department of electronics and information technology (Diety).

2. It has produced the first supercomputer assembled indigenously called Param Shivay.

Choose the correct statement

A. 1 only

B. 2 only

C. Both

D. None

Answer: C

Explanation: Both statements are correct.

National Supercomputing Mission (NSM) is being implemented and steered jointly by the Department of Science and Technology (DST) and the

Department of Electronics and Information Technology (DeitY). Implemented by the Centre for Development of Advanced Computing (C-DAC), Pune and the Indian Institute of Science (IISc), Bengaluru.

Achievements:

The first supercomputer assembled indigenously, called Param Shivay, was installed in IIT (BHU) and was

inaugurated by the Prime Minister. Similar systems Param Shakti and Param Brahma were installed at IIT-Kharagpur and IISER, Pune. They are equipped with applications from domains like Weather and Climate, Computational Fluid Dynamics, Bioinformatics, and Material science.

71. Recently a bio fortified crop by the name

of MACS 4028 was developed in India, which of the following corresponds to the same

Page 53: Free . Regular . Quality€¦ · 1. Currency notes are legal tenders 2. Currency notes are unlimited legal tenders 3. Currency notes are guaranteed by the Central Government 4. Currency

www.YouTube.com/SleepyClasses

www.sleepyclasses.com/

A. Wheat

B. Rice

C. Maize

D. Ragi

Answer: A

Explanation: Scientists of ARI, Pune develop biofortified, high protein wheat variety- MACS 4028.

It is a semi-dwarf variety. It has shown superior and stable yielding ability.

It is resistant to stem rust, leaf rust, foliar aphids, root aphids, and brown wheat mite.

72. Consider the following details with respect to the scheme SATHI (Sophisticated Analytical and technical help institute)

1. It has been launched by the Ministry of HRD.

2. It aims to address the need for building shared, professionally managed and strong S&T infrastructure in the country which is readily accessible to academia, start-ups, manufacturing, industry and R&D labs, etc.

Choose the correct answer

A. 1 only

B. 2 only

C. Both

D. None

Answer: B

Explanation:

To address the need for building shared, professionally managed and strong Science and Technology infrastructure in the country which is readily accessible to academia, start-

ups, manufacturing, industry and R&D labs, the Department of Science & Technology has launched a unique scheme called Sophisticated Analytical &

Technical Help Institutes (SATHI)it has been launched by the department of science and technology.

These Centres are expected to house major analytical instruments to provide common services of high-end analytical testing, thus avoiding duplication and reduced dependency on foreign sources. These would be operated with a transparent, open access policy.

73. Consider the statements with respect to the process of “Reverse Osmosis”

1. It involves ‘a solvent (such as water) naturally moving from an area of low solute concentration, through a membrane, to an area of high solute concentration.

2. The problem associated with it is the deposition of brine.

3. Water purifiers use the above process of reverse osmosis.

Choose the correct answer

A. 1 and 2

B. 2 and 3

C. 1 and 3

D. All of the above

Answer: B

Explanation:

1st statement defines osmosis and not reverses osmosis.

Reverse osmosis - A reverse osmosis system applies an external pressure to reverse the natural flow of solvent and so seawater brackish water is pressurized against one surface of the membrane, causing salt-depleted water

Page 54: Free . Regular . Quality€¦ · 1. Currency notes are legal tenders 2. Currency notes are unlimited legal tenders 3. Currency notes are guaranteed by the Central Government 4. Currency

www.YouTube.com/SleepyClasses

www.sleepyclasses.com/

to move across the membrane, releasing clean water from the low-pressure side’.

2nd and 3rd statement is correct.

74. Consider the following points with respect to NASA Voyager spacecraft

1. It is the only probe ever to study Neptune and Uranus during planetary flybys.

2. It is the second manmade object to leave the heliosphere.

3. It is the only spacecraft to have visited all the 4 gas giant planets.

Choose the correct answer

A. 1 and 2

B. 2 and 3

C. 1 and 3

D. All of the above

Answer: D

Explanation:

All of the above points are correct. NASA has managed to fix its Voyager-2 probe remotely, almost 11.5 billion miles away from its location. Accomplishments so far:

Voyager 2 is the only probe ever to study Neptune and Uranus during planetary flybys.

It is the second man-made object to leave the heliosphere.

Voyager 2 is the only spacecraft to have visited all four gas giant planets — Jupiter, Saturn, Uranus, and Neptune — and discovered 16 moons, as well as phenomena like Neptune’s mysteriously transient Great Dark Spot, the cracks in Europa’s ice shell, and ring features at every planet.

75. Consider the following points with respect to Aditya’s L1 mission

1. It is India’s first solar mission.

2. It will be launched using the GSLV in XL configuration.

3. The space-based observatory will have seven payloads.

Choose the correct answer

A. 1 and 2

B. 2 and 3

C. 1 and 3

D. All of the above

Answer: C

Explanation: Aditya- L1 mission is India’s first solar mission.

It will be launched using the Polar Satellite Launch Vehicle (PSLV) in XL configuration.

The space-based observatory will have seven payloads (instruments) on board to study the Sun’s corona, solar emissions, solar winds and flares, and Coronal Mass Ejections (CMEs), and will carry out round-the-clock imaging of the Sun.

76. Which one of the following became a part of China in 1997 following the principle of “one country, two systems”?

A. Tibet

B. Hong kong

C. Xinjiang

D. Taiwan

Answer: B

Explanation:

This question was asked in order to confuse you since currently, Taiwan is

Page 55: Free . Regular . Quality€¦ · 1. Currency notes are legal tenders 2. Currency notes are unlimited legal tenders 3. Currency notes are guaranteed by the Central Government 4. Currency

www.YouTube.com/SleepyClasses

www.sleepyclasses.com/

in use, over the avoidance of reference by WHO officials over the COVID-19 pandemic outbreak.

On the other hand, In Hong Kong, Xinjiang, Tibet & Taiwan, China follows a different politico-economic system than Mainland hence it is often referred to as one country two systems.

77. Consider the following statements about

ITBP – Indo Tibetan Border Police ITBP was raised in 1962.

1. It was basically a mountain raised forces.

2. It replaced Assam Rifles in Sikkim and Arunachal Pradesh in 2004-2005 for border guarding duty

Choose the correct answer

A. 1 and 2

B. 2 and 3

C. 1 and 3

D. All of the above

Answer: D

Explanation: Multiple Choice Type Question: ** Explanation Not Available **

78. India has signed a Comprehensive Economic Partnership Agreement (CEPA) with

1. USA

2. Singapore

3. Japan

Choose the correct answer

A. 1 and 2

B. 2 and 3

C. 1 and 3

D. All of the above

Answer: B

Explanation: India does not have any CEPA with USA.A comprehensive economic partnership agreement (CEPA) is a free trade agreement between two countries.

79. Consider the following statements with respect to Hansen’s disease

1. It is caused by bacteria Mycobacterium

laprae and mycobacterium lepromatosis.

2. It is non curable in nature.

3. It is generally common in poor people and transmitted by respiratory droplets.

Choose the correct answer

A. 1 and 2

B. 2 and 3

C. 1 and 3

D. All of the above

Answer: C

Explanation: This disease is also known by the name of leprosy.Also it is curable in nature with proper treatment.

80. Hodeidah, a city in new recently, is

located in which of the following countries?

A. Syria

B. Iraq

C. Iran

D. Yemen

Answer: D

Explanation:

UN-appointed India’s Lieutenant General (Retd.) Abhijit Guha as the

Page 56: Free . Regular . Quality€¦ · 1. Currency notes are legal tenders 2. Currency notes are unlimited legal tenders 3. Currency notes are guaranteed by the Central Government 4. Currency

www.YouTube.com/SleepyClasses

www.sleepyclasses.com/

head of the UN mission in Hodeidah, Yemen.

A Hodeidah Agreement was also signed between the Houthis and the government forces in 2018 to withdraw from the port city to allow vital humanitarian aid

81. Which of the following statement(s) is/are correct in the context of Chandrayaan 2

1. The Pragyan rover was designed to communicate with the Vikram lander, the lander with the orbiter and the orbiter with the Earth-based station

2. The lander was designed to function for 1 lunar day

Choose the correct statement

A. 1 only

B. 2 only

C. Both 1 and 2

D. Neither 1 nor 2

Answer: B

Explanation:

At the time of launch, the Chandrayaan 2 Orbiter will be capable of communicating with Indian Deep Space Network (IDSN) at Byalalu as well as the Vikram Lander. The mission life of the Orbiter is one year and it will be placed in a 100X100 km lunar polar orbit.

The Lander of Chandrayaan 2 is named Vikram after Dr. Vikram A Sarabhai, the Father of the Indian Space Programme. It is designed to function for one lunar day, which is equivalent to about 14 Earth days. Vikram has the

capability to communicate with IDSN at Byalalu near Bangalore, as well as with the Orbiter and Rover. The

Lander is designed to execute a soft landing on the lunar surface.

Chandrayaan 2's Rover is a 6-wheeled robotic vehicle named Pragyan, which translates to 'wisdom' in Sanskrit. It can travel up to 500 m (½-a-km) and leverages solar energy for its functioning. It can only communicate with the Lander

82. Which of the following best describes the term ‘Paraquat’?

A. It is a tribe that can recall the name of their ancestors from 20 generations

B. Name of an iceberg which recently broke off in Antarctica

C. A herbicide which doesn’t have an antidote

D. Missiles used to attack oil facilities in Saudi Arabia

Answer: C

Explanation:

Galo community in Arunachal Pradesh

is a tribe that can recall the name of their ancestors from 20 generations.

Paraquat is a toxic chemical that is widely used as an herbicide (plant killer), primarily for weed and grass control.

83. Consider the following statement about the Harappan Agriculture system

1. The Bull was known to the people of Harappan Civilization

2. Evidence of plough field has been found at Harappan sites

3. Harappan agriculture was dependent on monsoon and no irrigation facility was required

Page 57: Free . Regular . Quality€¦ · 1. Currency notes are legal tenders 2. Currency notes are unlimited legal tenders 3. Currency notes are guaranteed by the Central Government 4. Currency

www.YouTube.com/SleepyClasses

www.sleepyclasses.com/

Choose the correct answer from the following code:

A. 1 and 2 Only

B. 2 and 3 Only

C. All of the above

D. 1 and 3 Only

Answer: A

Explanation:

Statement 1 is correct: Representations on seals and terracotta sculpture indicate that the bull was known, and archaeologists extrapolate from this that oxen were used for ploughing.

Statement 2 is correct: Terracotta models of the plough have been found at sites in the Cholistan and at Banawali (Haryana). Archaeologists have also found evidence of a ploughed field at Kalibangan (Rajasthan), associated with Early Harappan levels. The field had two sets of furrows at right angles to each other, suggesting that two different crops were grown together.

Statement 3 is incorrect: Most Harappan sites are located in semi-arid lands, where irrigation was probably required for agriculture. Traces of canals have been found at the Harappan site of Shortughai in Afghanistan, but not in Punjab or Sind.

84. Consider the following about Mauryan Empire:

1. There was a uniform administrative system throughout the length and breadth of the Mauryan Empire

2. Asoka used the message of dhamma as

a political tool to keep the large Mauryan Empire together

Choose the correct answer from the following code:

A. 1 Only

B. Both 1 and 2

C. 2 Only

D. None of the above

Answer: C

Explanation:

Statement 1 is incorrect: The Mauryan

Empire was spread from northwest Frontier Provinces of Pakistan to Andhra Pradesh, Orissa, and Uttaranchal in India. Historians have increasingly come to realize that this is unlikely. The regions included within the empire were just too diverse. Imagine the contrast between the hilly terrain of Afghanistan and the coast of Orissa.

Statement 2 is correct: Asoka also tried to hold his empire together by propagating dhamma, the principles

of which, as we have seen, were simple and virtually universally applicable. This, according to him, would ensure the well-being of people in this world and the next. Special officers, known as the dhamma mahamatta, were appointed to spread the message of dhamma.

85. Arrange the following in chronological order with respect to their date of arrival in India?

1. Al-Biruni

2. Francois Bernier

3. Ibn Battuta

4. Marco Polo

Choose the correct answer from the following code:

Page 58: Free . Regular . Quality€¦ · 1. Currency notes are legal tenders 2. Currency notes are unlimited legal tenders 3. Currency notes are guaranteed by the Central Government 4. Currency

www.YouTube.com/SleepyClasses

www.sleepyclasses.com/

A. 1<2<3<4

B. 1<4<3<2

C. 3<4<2<1

D. 3<2<4<1

Answer: B

Explanation:

Tenth-eleventh centuries 973-1048 Muhammad ibn Ahmad Abu Raihan al-Biruni (from Uzbekistan)

Thirteenth-century 1254-1323 Marco Polo (from Italy)

Fourteenth-century 1304-77 Ibn Battuta (from Morocco)

Seventeenth-century 1620-88 Francois Bernier (from France)

86. Consider the following statements

1. Muhammad bin Tughlaq introduced the system of dhag and hullia as a part of military reform.

2. Alauddin Khalji created Diwan-i-Khairat to take care of orphans and widows

Choose the correct answer from the following code:

A. 1 Only

B. Both 1 and 2

C. 2 Only

D. None of the above

Answer: D

Explanation:

Statement 1 is incorrect: Alauddin Khalji introduced the system of dagh (branding of horses) and prepared huliya (descriptive list of soldiers). In order to ensure maximum efficiency, a

strict review of the army from time to time was carried out.

Statement 2 is incorrect: Firoz Tughlaq created Diwan-i-Khairat to take care of orphans and widows

87. Which of the following is not correct about dahsala system?

A. Revenue was fixed on the average

yield of the land assessed on the basis of the past 10 years.

B. The payment of revenue was generally made in cash.

C. The land was also divided into four categories Polaj, Milkiyat, Chachar, and Banjar.

D. The system was introduced by Todar Mal

Answer: C

Explanation: Land Revenue Administration:

Akbar made some experiments in the land revenue administration with the help of Raja Todar Mal. The land revenue system of Akbar was called Zabti or Bandobast system.

It was further improved by Raja Todar Mal. It was known as Dahsala System which was completed in 1580. By this system, Todar Mal introduced a uniform system of land measurement. The revenue was fixed on the average yield of land assessed on the basis of the past ten years.

The land was also divided into four categories – Polaj (cultivated every year), Parauti (once in two years), Chachar (once in three or four years) and Banjar (once in five or more years). The payment of revenue was made generally in cash.

Page 59: Free . Regular . Quality€¦ · 1. Currency notes are legal tenders 2. Currency notes are unlimited legal tenders 3. Currency notes are guaranteed by the Central Government 4. Currency

www.YouTube.com/SleepyClasses

www.sleepyclasses.com/

88. Consider the following statement about Asokan edict:

1. Asoka was the first ruler to speak directly to his people through the inscriptions

2. They are found only in India

3. The inscription was written in Brahmi script, Kharosthi script and Greek script

Choose the correct answer from the following code:

A. 1 and 2 Only

B. 2 and 3 Only

C. All 1, 2 and 3

D. 1 and 3 Only

Answer: D

Explanation:

Statement 1 is correct: Ashoka was the first king in Indian history who has left his records engraved on stones.

Statement 2 is incorrect: The Ashokan inscriptions are found at 47 places in different regions of India, Nepal, Pakistan, and Afghanistan.

Statement 3 is correct: Ashoka ‘s inscriptions were written in four different scripts, namely: Greek languages and scripts used in Afghanistan area; Aramaic languages and scripts used in west Asia; Prakrit language and Kharosthi script used in Pakistan area; and Prakrit language and Brahmi script used in the rest of the inscriptions.

89. The Fort William College at Calcutta was founded during the reign of which of the following?

A. Sir John Shore

B. Lord Wellesley

C. Sir George Barlow

D. Lord Hastings

Answer: B

Explanation: Lord Wellesley founded the

Fort William College at Calcutta. He opened College to train the Company’s servants in Calcutta. So, Known as the Father of Civil Services in India.

90. Consider the following statements about the Policy of Ring Fence:

1. It was aimed at creating buffer zones to defend the Company's frontiers.

2. The policy of subsidiary alliance was an extension of the Policy ring fence

3. Protection from Maratha and Afghans were the major reason for the Policy of Ring Fence.

Choose the answer from the following code:

A. 1 Only

B. 2 and 3 Only

C. 1 and 2 Only

D. All the above

Answer: D

Explanation:

This policy was reflected in Warren Hastings' wars against the Marathas and Mysore, and aimed at creating buffer zones to defend the Company's frontiers. The main threat was from the Marathas and Afghan invaders (the Company undertook to organize Awadh's defence to safeguard Bengal's security).

Wellesley's policy of subsidiary alliance was an extension of ring-fence—which sought to reduce states to a position of dependence on the British Government in India. Major Powers such as Hyderabad, Awadh

Page 60: Free . Regular . Quality€¦ · 1. Currency notes are legal tenders 2. Currency notes are unlimited legal tenders 3. Currency notes are guaranteed by the Central Government 4. Currency

www.YouTube.com/SleepyClasses

www.sleepyclasses.com/

and the Marathas accepted subsidiary alliance. Thus, British supremacy was established.

91. Arrange the following in chronological order:

1. Anglo-Nepal War

2. Fourth Mysore War

3. First Afghan War

4. Second Maratha War

Choose the answer from the following code:

A. 2<4<1<3

B. 4<2<3<1

C. 3<4<2<1

D. 1<2<3<4

Answer: A

Explanation:

Fourth Mysore War (1799).

Second Maratha War (1803-05).

Anglo-Nepal War (1814-16)

First Afghan War (1838-42).

92. Consider the following statement about

Revolt of 1857

1. The Revolt was sparked off by the episode of the greased cartridges

2. All the sections of Indian society supported the Revolt of 1857

3. Hindu-Muslim divide was the major cause of failure of the Revolt

Choose the correct answer from the following code:

A. 1 Only

B. 1 and 2 Only

C. All the above

D. 2 and 3 Only

Answer: A

Explanation:

Statement 1 is correct: The 1857 Revolt

was sparked off by the episode of the greased cartridges. The new Enfield rifle had been introduced for the first time in the Indian army. Its cartridges had a greased paper cover whose end had to be bitten off before the cartridge

was loaded into the rifle. The grease was composed of fat taken from beef and pig. The religious feelings of the Hindu and Muslim sepoys were terribly wounded. The sepoys believed that the government was deliberately trying to destroy their religious and cultural identity.

Statement 2 is incorrect: The first and foremost cause was that the Revolt

failed to embrace the whole of India. Different sections of society such as moneylenders, merchants and modern educated Indians were actually against the Revolt.

Statement 3 is incorrect: Hindu-Muslim unity was well displayed during the revolt of 1857. So much so that after the revolt, British Administrators took decisive steps to break this unity this culminated into a

partition of India in 1947.

93. Consider the following statements:

1. The book ‘Sarvajanik Satyadharma’

was written by Jyotiba Phule

2. Social Service League was founded by Narayan Malhar Joshi

3. The slogan ‘Back to the Vedas’ was given by Swami Vivekananda

Which of the statements given above is/are correct?

Page 61: Free . Regular . Quality€¦ · 1. Currency notes are legal tenders 2. Currency notes are unlimited legal tenders 3. Currency notes are guaranteed by the Central Government 4. Currency

www.YouTube.com/SleepyClasses

www.sleepyclasses.com/

A. 1 and 3 only

B. 2 only

C. 1 and 2 only

D. 1, 2 and 3

Answer: C

Explanation:

Statement 1 is correct: Jyotiba Phule belonged to the Mali (gardener) community and organized a powerful movement against upper caste domination and brahminical supremacy. Phule founded the Satyashodhak Samaj (Truth Seekers' Society) in 1873, with the leadership of the Samaj coming from the backward classes, Malis, Telis, Kunbis, Saris, and Dhangars. The main aims of the movement were (i) social service, and (ii) spread of education among women and lower caste people. Phule's works, Sarvajanik Satyadharma and Gulamgin, became sources of inspiration for the common masses.

Statement 2 is correct: Narayan Malhar Joshi founded the Social Service League in Bombay with an aim to secure for the masses better and reasonable conditions of life and work They organized many schools, libraries, reading rooms, day nurseries, and cooperative societies. Their activities also included police court agents' work, legal aid and advice to the poor and illiterate, excursions for slum dwellers, facilities for gymnasia and theatrical performances, sanitary work, medical relief, and boys' clubs and scout corps. Joshi also founded the All India Trade Union (1920).

Statement 3 is incorrect: The slogan ‘Back to the Vedas’ was given by Dayanand Saraswati. He took inspiration from the Vedas and considered them to be "India's Rock of

Ages", the infallible and the true original seed of Hinduism.

94. With reference to the Indian Councils Act of 1892, consider the following statements:

1. It introduced an element of election for the first time

2. The budget could be discussed and voted upon

3. It provided for the non-official majority in the Imperial Legislative Council

Which of the statements given above is/are correct?

A. 1 only

B. 1 and 2 only

C. 2 and 3 only

D. 1, 2 and 3

Answer: A

Explanation:

Statement 1 is correct: Number of additional members in Imperial Legislative Councils and the Provincial Legislative Councils was raised. In Imperial Legislative Council, now the governor-general could have ten to sixteen non-officials. Some of these additional members could be indirectly elected Thus an element of the election was introduced for the first time.

Statement 2 is incorrect: Budget could be discussed. However, it could not be voted upon, nor could any amendments be made to it.

Statement 3 is incorrect: The officials retained their majority in the council.

95. Consider the following statements:

Page 62: Free . Regular . Quality€¦ · 1. Currency notes are legal tenders 2. Currency notes are unlimited legal tenders 3. Currency notes are guaranteed by the Central Government 4. Currency

www.YouTube.com/SleepyClasses

www.sleepyclasses.com/

1. The demand of a full-fledged mass struggle with the goal of attaining Swaraj was approved by the Congress in its 1905 session

2. Swadeshi movement witnessed widespread participation from upper and middle-class Muslims

3. Traditional festivals were used to spread the propaganda of the Swadeshi movement

Which of the statements given above is/are incorrect?

A. 1 and 3 only

B. 1 and 2 only

C. 2 and 3 only

D. 1, 2 and 3

Answer: B

Explanation:

Statement 1 is incorrect: The militant nationalists led by Tilak, Lajpat Rai, Bipin Chandra Pal, and Aurobindo Ghosh wanted the movement to be taken outside Bengal to other parts of the country and go beyond a boycott of foreign goods to become a full-fledged political mass struggle with the goal of attaining swaraj. But the Moderates, dominating the Congress at that time,

were not willing to go that far. However, a big step forward was taken at the Congress session held at Calcutta (1906) under the presidentship of Dadabhai Naoroji, where it was declared that the goal of the Indian Congress was 'self-government or swaraj like the United Kingdom or the colonies.

Statement 2 is incorrect: Some of the Muslims participated in the Swadeshi movement— Barrister Abdul Rasul, Liaqat Hussain, Guznavi, Maulana Azad. But most of the upper and middle-class Muslims stayed away or,

led by Nawab Salimullah of Dacca, supported the partition on the plea that it would give them a Muslim-majority East Bengal.

Statement 3 is correct: Imaginative use of traditional popular festivals and, melas: The idea was to use such occasions as a means of reaching out to the masses and spreading political messages. For instance, Tilak's Ganapati and Shivaji festivals became a medium of swadeshi propaganda not only in western India, but also in Bengal. In, Bengal also, the traditional folk theatre forms were used for this purpose.

96. With reference to the revolutionary terrorist programme, consider the following statements:

1. Revolutionary terrorists aimed to create a violent mass revolution throughout the country.

2. Barrah dacoity was organized by Aurobindo Ghosh

3. Mitra Mela, a secret society, was organized by V.D Savarkar

Which of the statements given above is/are correct?

A. 1 and 2 only

B. 3 only

C. 2 and 3 only

D. 1, 2 and 3

Answer: B

Explanation:

Statement 1 is incorrect: Revolutionary terrorism was a by-product of the process of the growth of militant nationalism in India. It acquired a more activist form as a fallout of the Swadeshi and Boycott movement. The

Page 63: Free . Regular . Quality€¦ · 1. Currency notes are legal tenders 2. Currency notes are unlimited legal tenders 3. Currency notes are guaranteed by the Central Government 4. Currency

www.YouTube.com/SleepyClasses

www.sleepyclasses.com/

revolutionary terrorists considered but did not find it practical at that stage the options of creating a violent mass revolution throughout the country or, of trying to subvert the loyalties of the Army.

Statement 2 is incorrect: In 1908, Barrah dacoity was organized by Dacca Anushilan under Pulin Das.

Statement 3 is correct: V.D Savarkar and his brother organized Mitra Mela, a secret society, in 1899 which merged with Abhinav Bharat (after Mazzinni's 'Young Italy') in 1904.

97. With reference to Morley Minto reforms, consider the following statements:

1. It introduced non-official majority in the Imperial Legislative Council

2. It provided for direct elections to the provincial legislature

3. Separate electorates were provided for the Muslims

Which of the statements given above is/are incorrect?

A. 1 and 3 only

B. 2 only

C. 1 and 2 only

D. 1, 2 and 3

Answer: C

Explanation:

Statement 1 is incorrect: The number of elected members in the Imperial Legislative Council and the Provincial Legislative Councils was increased. In the Provincial Councils, the non-official majority was introduced, but since some of these non-officials were nominated and not elected, the overall non-elected majority remained.

In Imperial Legislative Council, the official majority was retained.

Statement 2 is incorrect: The elected members were to be indirectly elected. The local bodies were to elect an electoral college, which in turn would elect members of provincial legislatures, who in turn would elect members of the central legislature.

Statement 3 is correct: Besides separate electorates for the Muslims, representation in excess of the strength of their population was accorded to the Muslims. Also, the income qualification for Muslim voters was kept lower than that for Hindus.

98. With reference to the Ghadr programme,

consider the following statements:

1. Their aim was to bring simultaneous

revolt in all British colonies

2. It lacked an organized and sustained leadership

3. It preached militant nationalism with a completely secular approach

Which of the statements given above is/are correct?

A. 1 and 2 only

B. 1 and 3 only

C. 2 and 3 only

D. 1, 2 and 3

Answer: D

Explanation:

Statement 1 is correct: The Ghadar programme was to organize assassinations of officials, publish revolutionary and anti-imperialist literature, work among Indian troops stationed abroad, procure arms and bring a simultaneous revolt in all British colonies.

Page 64: Free . Regular . Quality€¦ · 1. Currency notes are legal tenders 2. Currency notes are unlimited legal tenders 3. Currency notes are guaranteed by the Central Government 4. Currency

www.YouTube.com/SleepyClasses

www.sleepyclasses.com/

Statement 2 is correct: The achievement of the Ghadar movement lay in the realm of ideology. It preached militant nationalism with a completely secular approach.

Statement 3 is correct: Militarily, Ghadar programme failed to achieve much because it lacked an organized and sustained leadership, underestimated the extent of preparation required at every level—organizational, ideological, financial and tactical strategic—and perhaps Lala Hardayal was unsuited for the job of an organizer

99. With reference to Home Rule League movement, consider the following statements:

1. Annie Besant set up her league before

Tilak

2. Formation of linguistic state was one of the demands of the league set-up by Tila

3. Anglo-Indians and non-brahmins from the south did not join the Home Rule League

Which of the statements given above is/are correct?

A. 1 and 3 only

B. 2 and 3 only

C. 1 and 2 only

D. 1, 2 and 3

Answer: B

Explanation:

Statement 1 is incorrect: Tilak's League was set up in April 1916 and was restricted to Maharashtra (excluding Bombay city), Karnataka, Central Provinces, and Berar. Besant's League was set up in September 1916 in

Madras and covered the rest of India (including Bombay city). It had 200 branches, was loosely organized as compared to Tilak's League and had George Arundale as the organizing secretary.

Statement 2 is correct: League set-up by Tilak demanded Swarajya, the formation of linguistic states and education in the vernacular languages.

Statement 3 is correct: The Home Rule agitation was later joined by Motilal Nehru, Jawaharlal Nehru, Bhulabhai Desai, Chittaranjan Das, Madan Mohan Malaviya, Mohammad Ali Jinnah, Tej Bahadur Sapru and Lala Lajpat Rai. Some of these leaders became heads of local branches. Many of the Moderate Congressmen who were disillusioned with. Congress inactivity and some members of Gokhale's Servants of India Society also joined the agitation. However, Anglo-Indians, most of the Muslims and nonbrahmins from the

South did not join as they felt Home Rule would mean the rule of the Hindu majority, mainly the high caste.

100. Which of the following demands were put forward by the Lucknow Pact?

A. Complete independence from the

British rule

B. All the members of the viceroy’s executive council should be Indians

C. Elected majority in the legislative councils

D. None of the above

Answer: C

Explanation:

The following demands were put forth by the Muslim League and Indian National Congress under the Lucknow Pact:

Page 65: Free . Regular . Quality€¦ · 1. Currency notes are legal tenders 2. Currency notes are unlimited legal tenders 3. Currency notes are guaranteed by the Central Government 4. Currency

www.YouTube.com/SleepyClasses

www.sleepyclasses.com/

The government should declare that it would confer self-government on Indians at an early date.

The legislative councils should be further expanded with an elected majority and more powers be given to them.

Half the members of the viceroy's executive council should be Indians